Final

अब Quizwiz के साथ अपने होमवर्क और परीक्षाओं को एस करें!

Jordan is 55 and wants to retire in 12 years. His family has a history of living well into their 90s. Therefore, he estimates that he will live to age 97. He currently has a salary of $100,000 and expects that he will need about 82% of that amount annually if he were retired. He can earn 9 percent in his portfolio and expects inflation to continue at 3 percent. Jordan currently has $325,000 invested for his retirement. His Social Security benefit in today's dollars is $30,000 per year at normal age retirement of age 67. How much does he need to save each year at year end to meet his retirement goals?

$ 9252 Given: Current age of Jordan is 55 years and he wishes to retire in 12 years. He estimates that he will live to age 97. Hence, his savings in the next 12 years should help him meet his retirement goals. He expects a salary of $ 100,000 per year and he will need 82% of the amount annually in case he retires. i.e. for 12 years, he will need $82000 annually during his retirement tenure for the next 30 years. He has made an investment of 325,000 which will give a return of 9% on his portfolio and inflation is at 3%.Hence, the returns he can expect is 325,000*0.09*0.03=$ 877.5 Social security benefit is $ 30,000 per year which Jordan will get as he retires. The salary amount will also be subject to inflation. Hence $ 18,000 that is needed for expenses at 3% inflation is $540 and expenses of $1800 Hence, the total amount that should be saved during his working period to meet his retirement goals is, 82,000+30,000+877.5+540+1854=$111024 Each year, the savings would be 111024/12=$ 9252

Jessica's employer matches each dollar Jessica contributes to her 403(b) plan each year. If Jessica elects to treat her $10,000 contribution to her 403(b) plan as a Roth contribution, how much can Jessica elect to treat of her employer's $10,000 contribution as a Roth contribution this year? Jessica is 35 years old and her AGI for the year will be $90,000.

$0

Kathy (age 55) is single, recently divorced, and has $250,000 in income this year. What is the most that Kathy can contribute to a Roth IRA for 2018?

$0

Robbin is married to Robert and they have one child Angel, age 14, who is in the 6th grade. Angel is a difficult child and she is cared for in the afternoon by the Sisters of Reformation, a group of Catholic nuns. Robbin pays $6,000 per year for the child care. Robbin's company has a dependent care assistance program. If Robbin makes the maximum use of the dependent care assistance program, how much can she exclude from her income if she files a joint return with Robert?

$0

Joyce and Melvin have been married for 30 years. In 2019, they received $22,000 of Social Security benefits and had $12,000 of interest income. What portion of the Social Security benefit is taxable?

$0. Rationale The lesser of: 50% of $22,000 = $11,000 Or 0.5 [$12,000 + 0.5 ($11,000) - $32,000] < 0 Since the answer calculated is less than $0, none of the Social Security benefits received by Joyce and Melvin are taxable.

Meredith is an employee of a large company. They are very interested in the betterment of the health of all employees. The company has a health facility on its premises for the exclusive use of its employees and their dependents. A comparable private health club membership at a public facility would cost $2,400 per year. How much, if any, must Meredith include in her gross income if her 10-year old daughter uses the facilities for one-half of the year?

$0. Rationale The value of a health facility provided by the employer, on the employer's premises, and solely for the use of employees and their dependents is excluded from gross income by the employee. It does not appear that the 2017 TCJA impacted this fringe benefit.

Natalie is a secretary at JKL Law Firm. JKL provides her with free sodas at her discretion. Natalie estimates that she drinks $20 worth of sodas per month. How much must Natalie include in her annual gross income related to the sodas?

$0. Rationale The value of employer-provided de minimis fringe benefits may be excluded from an employee's gross income. However, after 2017, the employer's deduction for the sodas is limited to 50% due to TCJA 2017.

Jim, who is age 39, converts a $74,500 Traditional IRA to a Roth IRA in 2019. Jim's adjusted basis in the Traditional IRA is $10,000. He also makes a contribution of $6,000 to a Roth IRA in 2019 for the tax year 2019. If Jim takes a $4,000 distribution from his Roth IRA in 2020 when the account is worth $100,000, how much total federal income tax, including penalties, is due as a result of the distribution assuming his 2020 federal income tax rate is 24 percent?

$0. Rationale Any amount distributed from an individual's Roth IRA is treated as made in the following order (determined as of the end of a taxable year and exhausting each category before moving to the following category): • From regular contributions; • From conversion contributions, on a first-in-first-out basis; and • From earnings. Jim's $4,000 distribution comes from the "contribution" layer, and is therefore, not subject to tax or penalty. All distributions from all of an individual's Roth IRAs made during a taxable year are aggregated. The 10% additional tax under IRC § 72(t) applies to any distribution from a Roth IRA includible in gross income. The 10% additional tax under IRC §72(t) also applies to a nonqualified distribution, even if it is not then includible in gross income, to the extent it is allocable to a conversion contribution and if the distribution is made within the five-taxable-year period beginning with the first day of the individual's taxable year in which the conversion contribution was made.

Kathryn taught at Fun Academic University for 25 years and is filing for Social Security retirement benefits this year, when she turns age 72. Her PIA is $1,000 per month as adjusted for inflation. How much in benefits will she receive assuming her full retirement age was 66?

$1,320. Rationale Delayed benefits will increase at 8% per year of delay up until age 70. Therefore, her benefit will be 32% higher than her PIA. She has no increase for delaying beyond age 70 and the 8% increase is not compounded.

Tracy, age 46, is a self-employed financial planner and has Schedule C income from self-employment of $56,000. He has failed to save for retirement until now. Therefore, he would like to make the maximum contribution to his profit sharing plan. How much can he contribute to his profit sharing plan account for 2018?

$10,409.

Tracy, age 46, is a self-employed financial planner and has Schedule C income from self-employment of $56,000. He has failed to save for retirement until now. Therefore, he would like to make the maximum contribution to his profit sharing plan. How much can he contribute to his profit sharing plan account for 2019?

$10,409. Rationale $56,000 Schedule C net income - 3,956 (less 1/2 self-employment taxes at 15.3% x 0.9235) $52,044 Net self-employment income x 0.20 (0.25 ÷ 1.25) $10,409 Keogh profit sharing contribution amount

For the year 2019, Katy (age 35) and Stefen (age 38), a married couple, reported the following items of income: Katy Stefen Total Wages $50,000 - $50,000 Dividend Income $2,000 $1,200 $3,200 Cash won from lottery $500 $500 $52,000 $1,700 $53,700 Katy is covered by a qualified plan. Stefen does not work; he makes his own wine and samples it most of the day. Assuming a joint return was filed for 2019, what is the maximum tax deductible amount that they can contribute to their IRAs?

$12,000. Rationale Because their income is less than the limit for joint income tax filers ($103,000 for 2019), they can contribute and deduct $12,000 for 2019.

Mary and Ed, both age 33, are married, not covered by a qualified plan, and file a joint tax return. They have AGI of $164,000. Mary's mother contributed $2,000 to a Coverdell Education Savings Account for each of their two children. What is the most that Mary and Ed can contribute in total together to a Traditional IRA for 2019?

$12,000. Rationale The maximum contribution to Traditional and Roth IRAs is a total of $6,000 per person (who has not attained age 50) for 2019. The limits for the Coverdell are not related to traditional or Roth IRAs.

Parker is a highly skilled sales person at Byberry, which is a 30-year-old company that has grown significantly in terms of revenue and product offerings. It sponsors a pension plan that provides a benefit of 2% times the years of participation times the average of the final three years of salary. Parker has worked for Byberry for the last 30 years and earned $200,000 two years ago, $150,000 last year, and $250,000 this year. If he is retiring this year, how much should he expect to receive as a pension benefit?

$120,000

QP, LLC sponsors a defined contribution plan. Joe, age 52, has compensation of $295,000 for the year. QP has made a 17% profit sharing plan contribution on Joe's behalf. There were no forfeitures allocated to participants. How much can Joe defer into his 401(k) (CODA plan) to maximize his annual contributions to the qualified plan for 2018? a. $4,850 b. $8,250 c. $14,250 d. $18,500 e. $24,500

$14,250

Jose Sequential, age 70½ in October of this year, worked for different company (A) prior to his current employer (B). His qualified balance at the end of last year for Company A was $400,000; the balance at Company B was $150,000. What is his required minimum distribution for the current year from these plans that he must take by April 1st next year? Life expectancy tables are 27.4 for age 70 and 26.5 for age 71.

$14,599

ABC Inc. sponsors a defined contribution plan. Seth, age 39, has compensation of $150,000 for the year. ABC has made a $20,000 profit sharing plan contribution for Seth and $5,000 of plan forfeitures were allocated to Seth's profit sharing plan during the year. How much can Seth defer into his CODA plan (401(k) plan) for 2018?

$18,500

WHR, LLC sponsors a defined contribution plan. Vaughn, age 44, has compensation of $160,000 for the year. WHR has made a $25,000 profit sharing plan contribution on Vaughn's behalf and $5,000 of plan forfeitures were allocated to Vaughn's profit sharing plan during the year. How much can Vaughn defer into his CODA plan (401(k)) to maximize his annual contributions to the qualified plan for 2018?

$18,500

Nick, who is age 45, operates a landscaping business and is self-employed. He has an assistant, Louis, who has worked with him for five years. Nick is establishing a SEP for 2019 and is willing to make a contribution of 25 percent of Louis's salary to the SEP. If Nick earns $100,000 after paying Louis, his expenses, and the contribution to Louis's SEP, what is the most that he can contribute to the SEP for himself?

$18,587. Rationale Choice a is correct. $100,000 less [$100,000 x 0.9235 x 0.153 x ½] x 20% (0.25/1.25) = $18,587. Choice b left out the self employment income. Choice c is incorrect because it does not adjust the 25% to 20%. Choice d is incorrect as it multiplies by 25% instead of 20%.

What is the maximum catch-up contribution for 2019 under the 457(b) plan "Final 3-Year" rule?

$19,000. Rationale The final 3-year catch-up is $19,000 for 2019. The final 3-year catch-up cannot be combined with the age 50 or over catch-up.

Bob works for New Orleans Museum of Art, which sponsors a 403(b) plan. If Bob is 45 years old and has worked at the museum for the last 20 years, what is his maximum elective deferral for 2019?

$19,000. Rationale The salary reduction for 2019 is $19,000. An additional catch-up contribution of $6,000 is allowed for individuals who have attained age 50. The other type of catch-up contribution is not available to employees of employers such as museums.

Kathy (age 55) is single and was divorced from her husband in 2017. She has received the following items of income this year: Pension annuity income from QDRO $21,000 Interest and dividends $5,000 Alimony $1,000 W-2 Income $1,200 What is the most that Kathy can contribute to a Roth IRA for 2019?

$2,200. Rationale Contributions to Roth IRAs, as well as traditional IRAs, are limited to the lesser of earned income or $6,000 for 2019. Kathy has earned income of $2,200 from the alimony and W-2 income she received. Thus, she is limited to a contribution of $2,200. The other $26,000 of income is not earned income and therefore is unavailable for contributions to any IRA. An additional catch-up contribution of $1,000 for 2019 is permitted for individuals who have attained age 50 by the close of the tax year. Her total remains at $2,200 because that is all the earned income she has. Note: Alimony resulting from divorce agreements after 2018 is not income or earned income (TCJA 2017).

Colin is 40 years old and wants to retire in 27 years. His family has a history of living well into their 90s. Therefore, he estimates that he will live to age 95. He currently has a salary of $150,000 and expects that he will need about 75% of that amount annually if he were retired. He can earn 8 percent from his portfolio and expects inflation to continue at 3 percent. Some years ago, he worked for the government and expects to receive an annuity that will pay him $20,000 in today's dollars per year beginning at age 67. The annuity includes a cost of living adjustment, which is equal to inflation. Colin currently has $200,000 invested for his retirement. His Social Security benefit in today's dollars is $30,000 per year at normal age retirement of age. How much does he need to accumulate at age 67 exclusive of his pension and Social Security benefits?

$2.2 mil

Jake has worked for GT for the last 20 years and been a participant in its defined benefit plan. In the last ten years, his salary has increased significantly. Over the last ten years, his compensation was $290,000, $100,000, $120,000, $100,000, $240,000, $200,000, $160,000, $180,000, $150,000, and $210,000. In 2018, what is the most that he could receive as a pension payment? a. $200,000. b. $220,000. c. $246,667. d. $275,000.

$200,000. The maximum distribution from a defined benefit plan in 2018 is the lesser of $220,000 or the average of the three highest consecutive years of compensation. The average of the three highest consecutive years of compensation equals $200,000.

Kim Cat, age 42, earns $300,000 annually as an employee for CTM, Inc. Her employer sponsors a SIMPLE retirement plan and matches all employee contributions made to the plan dollar-for-dollar up to 3% of covered compensation. What is the maximum contribution (employer and employee) that can be made to Kim's SIMPLE account in 2019?

$22,000. Rationale The maximum total contribution is $22,000. ($13,000 maximum employee contribution for 2019 + $9,000 employer match). The maximum employee contribution for 2019 is $13,000. The employer has chosen to make matching contributions up to 3% of compensation (the SIMPLE maximum). Therefore, the employer can make a contribution of up to $9,000 ($300,000 compensation x 3%). Note that the annual compensation limit of $280,000 (2019) does not apply with the SIMPLE match (but does apply with a non-elective contribution).

Jim, a participant in the Zappa retirement plan, has requested a second plan loan. Jim's vested account balance is $80,000. He borrowed $27,000 eight months ago and still owes $18,000 on that loan. How much can he borrow as a second loan?

$22,000. Rationale He can borrow the lesser of $50,000 or half of the vested account balance. The $50,000 must be reduced by the highest outstanding balance in the last twelve months - $23,000. Half of the vested account balance ($40,000) less the outstanding loan of $18,000 equals $22,000.

Bill is single, retired, and received Social Security retirement benefits of $28,000 this year. His other income consists of his monthly $5,000 pension payment from his prior employer; $5,000/yr in tax-exempt municipal bond interest; and $1,000/yr in dividends. How much of Bill's Social Security Benefit will be subject to income tax?

$23,800.

Dwayne Barnum, age 52, earns $300,000 annually as an employee for DCH, Partners. His employer sponsors a SIMPLE retirement plan and matches all employee contributions made to the plan dollar-for-dollar up to 3% of covered compensation. What is the maximum contribution (employer and employee) that can be made to Dwayne's SIMPLE account in 2018?

$24,500

An employee has both a 403b and 457b plan at work. What is the combined total contribution possible to these plans in 2018 for an employee aged 55?

$24,500 combined

Dr. Wood is able to rollover amounts in the traditional deferral account for the 457 plan into the Roth account without terminating employment. Rationale Choice b is not correct as forfeitures cannot go into a 457 plan. Also, it is unlikely there are any forfeitures. Choice c is not correct as 403(b) plans cannot be invested in individual securities. Choice d is incorrect as the assets in a public 457 plan are protected by trust.

$25,000. Rationale Henry is not within three years of the plan's normal retirement age and therefore can only defer the normal $19,000 available plus the $6,000 catch-up for those participants age 50 and over.

What is the maximum elective deferral contribution to an eligible governmental 457(b) plan for 2019?

$25,000. Rationale The maximum deferral is $19,000 for 2019. The maximum catch-up contribution is $6,000 for those participants age 50 and over. Therefore, the maximum elective deferral contribution including the catch-up is $25,000.

In 2018, eGadget Corporation has covered compensation as a company totaling $1,000,000. The employees of eGadget make employee deferrals into the company's qualified plan of $125,000. What is the total amount the eGadget can take as an employer deduction for employer contributions made to its qualified plan?

$250,000

Jean works for A&R Law Firm, which has a qualified transportation and parking benefits program. A&R provides for $265 for qualified parking per month. How much of the qualified parking is excluded from Jean's gross income and how much is deductible by A&R?

$265 excluded and $0 deductible by A&R. Rationale TCJA 2017 disallows a deduction for expenses associated with providing any qualified transportation fringe to employees of a taxpayer after 2017. However, these expenses are not included in income for the employee. The exception is for safety reasons.

Ben Reynolds, age 63 in 2019, is planning for retirement at normal retirement age of 65 from the Salt Lake City coroner's office, which has a 457(b) plan. Ben has an unused deferral amount of $9,000. Ben has compensation of $128,000 per year as a mortician/autopsy specialist. Ben wants to know the maximum amount he can defer in 2019 in the 457(b) plan.

$28,000. Rationale He can defer $19,000 + $9,000, the unused deferral, for a total of $28,000. He cannot combine the three-year rule and the over 50 rule.

Tracy, age 46, is a self-employed financial planner and has Schedule C income from self-employment of $56,000. He has failed to save for retirement until now. Therefore, he would like to make the maximum contribution to his 401(k) profit sharing plan. What number is closest to how much can he contribute to his account for 2018?

$28,900

Jane is covered by a $90,000 group-term life insurance policy, her daughter is the sole beneficiary. Jane's employer pays the entire premium for the policy; the uniform annual premium is $0.60 per $1,000 per month of coverage. How much, if any, is W-2 taxable income to Jane resulting from the insurance?

$288. Rationale $50,000 of group-term life insurance is nontaxable. $90,000 - 50,000 = 40,000 x $0.60 per thousand x 12 = $288 taxable.

(2pts) Charlie would like to retire in 11 years at the age of 66. He would like to have sufficient retirement assets to allow him to live on 90% of his current income adjusted for inflation. He expects to receive $24,000 per year from Social Security, which will increase with inflation. Charlie assumes that he will earn 9% annually on his investments, that inflation will be 4% per year and that he will live to be 101 years old. If Charlie currently earns $150,000, how much does he need at retirement?

$3,005,083

On January 5, Cindy, age 39, withdrew $42,000 from her qualified plan. Cindy had an account balance of $180,000 and an adjusted basis in the account of $30,000. Calculate any early withdrawal penalty.

$3,500. Rationale $30,000/$180,000 = 0.1667 exclusion. $42,000 x 0.1667 = $7,000. $42,000 - $7,000 = $35,000 x 0.10 = $3,500.

ABC Corp. provides employees with discounts on the flat panel televisions they manufacture. The discounts were established using a length of service and employee status methodology. Length of Service Employee Discount Officer Discount 1-5 years 10% 20% > 5-10 years 15% 30% > 10 years 20% 40% The gross profit percentage for ABC is 40%. George is an officer employee who has been with ABC Corp. for 13 years. For Christmas this year, George bought a 56 inch flat panel television that retails for $8,800 and he received a discount appropriate to the schedule discounts listed above. For this year, how much, if any, does George have to include in gross income as a result of this transaction? A)

$3,520. Rationale The plan is clearly discriminatory; therefore, George, who is an officer, must include the entire discount of $3,520 ($8,800 x 0.40).

Axe company sponsors a 401(k) profit sharing plan with no employer match, but the company did make employer contributions because the plan was top-heavy. Jack quit today after six years working for Axe and has come to you to determine how much of his retirement balance he can take with him. The plan uses the least generous graduated vesting schedule available. What is Jack's vested account balance? (earnings $4,000 &$5,000)

$30,000

Nancy, age 70 on February 2, 2019, had the following account balances in a qualified retirement plan. 12/31/2018 $500,000 12/31/2019 $478,000 12/31/2020 $519,000 12/31/2021 $600,000 Assuming that Nancy is retired and has never taken a distribution prior to 2020, what is the total amount of minimum distribution required in 2020? Life expectancy factors according to the uniform life table are 27.4 for a 70 year old and 26.5 for a 71 year old.

$36,286. Rationale For 2019, look back to 2018: $500,000 ÷ 27.4 = $18,248 For 2020, look back to 2019: $478,000 ÷ 26.5 = $18,038 $18,248 + $18,038 = $36,286 She must take a distribution for 2019 and 2020. However, she can wait to take the 2019 distribution until April 1, 2020 in which case she has 2 distributions in 2020.

Nancy, age 70 on February 2, 2019, had the following account balances in a qualified retirement plan. 12/31/2018 $500,000 12/31/2019 $478,000 12/31/2020 $519,000 12/31/2021 $600,000 Assuming that Nancy is retired and has never taken a distribution prior to 2020, what is the total amount of minimum distribution required in 2020? Life expectancy factors according to the uniform life table are 27.4 for a 70 year old and 26.5 for a 71 year old

$36,286. Rationale For 2019, look back to 2018: $500,000 ÷ 27.4 = $18,248 For 2020, look back to 2019: $478,000 ÷ 26.5 = $18,038 $18,248 + $18,038 = $36,286 She must take a distribution for 2019 and 2020. However, she can wait to take the 2019 distribution until April 1, 2020 in which case she has 2 distributions in 2020.

James, age 58, has compensation of $150,000 and wants to defer the maximum to his public 457(b) plan. The normal retirement age for his plan is age 60. How much can he defer in 2019 if he has an unused deferral amount of $60,000 from age 40 to age 49?

$38,000. Rationale He can contribute $38,000 (2 x $19,000). He must be within three years of retirement and have unused deferral. Note that since he used the final 3-year catch-up, he cannot use the over 50 catch-up.

Diggs is a 47-year-old executive who earns $315,000 from his job at Acme Arrows (AA) and contributes the maximum amount to the AA 401(k) plan. He wants to make a contribution to a Roth IRA for the current year, but his compensation is over the income limit. He decides he wants to fund a Roth IRA by using the backdoor Roth technique. Assume that Diggs has a traditional IRA with a balance of $24,000 that was funded entirely with pre-tax contributions. If Diggs contributes $6,000 to a traditional IRA and then converts $6,000 to a Roth IRA, how much income will he have to pick up as a result of the conversion?

$4,800. Rationale The backdoor Roth technique is less effective when the taxpayer has funds in a traditional IRA as all IRA amounts must be aggregated. Thus, Diggs must pick up $4,800 into income [1 - ($6,000 / ($6,000 + $24,000))] x $6,000.

Jose Sequential, age 70½ in October of this year, worked for several companies over his lifetime. He has worked for the following companies (A-E) and still has the following qualified plan account balances at those companies. Company Jose's Account Balance A $250,000 B $350,000 C $150,000 D $350,000 E $200,000 Jose is currently employed with Company E. What, if any, is his required minimum distribution for the current year from all plans? Life expectancy tables are 27.4 for age 70 and 26.5 for age 71.

$40,146. Rationale Jose is required to take a minimum distribution for the years in which he is 70½ from each qualified plan, except from his current employer ($1,100,000 ÷ 27.4 = $40,146). He can delay the payment until April 1 of next year, but the question asks for the distribution required for the current year. Note: He must take from each account. He cannot take $40,146 from one account as he could if A-D were IRAs.

Vance is 70½ on April 1 of the current year and must receive a minimum distribution from his traditional IRA. The account balance had a value of $423,598 at the end of last year. The distribution period for a 70 year old is 27.4, and for a 71 year old it is 26.5. If Vance takes a $15,000 distribution next April 1st, what is the amount of the minimum distribution tax penalty associated with his first year's distribution?

$492

Gerry is 70½ on April 1 of the current year and must receive a minimum distribution from his qualified plan. The account balance had a value of $423,598 at the end of last year. The distribution period for a 70 year old is 27.4, and for a 71 year old it is 26.5. If Gerry takes a $15,000 distribution next April 1st, what is the amount of the minimum distribution tax penalty associated with his first year's distribution?

$492. Rationale The required minimum distribution for Gerry is $15,985 ($423,598 divided by 26.5) because he is 71 years old as of December 31 of the current year. Gerry only took a distribution of $15,000, therefore, the minimum distribution penalty (50%) would apply to the $985 balance. Therefore, the minimum distribution penalty is $492 (50% of the $985).

Charles, a single 29-year-old CEO of a technology start-up company, earns a $2 million base salary with a $400,000 bonus. He is not a participant in any retirement plans at work. What is the maximum deductible IRA contribution Charles can make during 2018?

$5,500

Robin and Robbie, both age 45, are married and filed a joint return for 2018. Robbie earned a salary of $100,000 in 2018 and is covered by his employer's 401(k) plan. Robbie and Robin earned interest of $30,000 in 2018 from a joint savings account. Robin is not employed, and the couple had no other income nor any above the line tax deductions. On April 15, 2019, Robbie contributed $5,500 to an IRA for himself and $5,500 to an IRA for Robin. The maximum allowable IRA deduction on the 2018 joint return is:

$5,500

Dr. Means has taught accounting at FAU for the last 30 years and is expected to retire next year, at age 65. FAU sponsors a 403(b) plan and a 457(b) governmental plan. She has been diligent and has always contributed the maximum amounts to each of the plans. If her salary is $100,000, how much can she contribute in total to both plans in 2019?

$50,000. Rationale She can defer the maximum of $19,000 to each plan and the additional over the age of 50 catch up of $6,000 for both plans, which totals $50,000. She cannot use the other catch up provisions for 403(b) plans or 457 plans since she has maximized her deferrals in the past.

Cavin sells stock several years after he received it as a distribution from a qualified stock bonus plan. When the stock was distributed, he had a net unrealized appreciation of $7,500. Cavin also had ordinary income from the distribution of $29,000. The fair market value of the stock and the sales price at the time of sale was $81,000. How much of the sale price will be subject to long-term capital gain treatment?

$52,000 Selling price- Ordinary Income = $81,000-$29,000=$52,000

For 2019, what is the maximum amount that can be contributed to a SEP?

$56,000. Rationale For 2019, the maximum contribution for an individual to a SEP is the lesser of: 25% of compensation (compensation maximum is $280,000), or $56,000. Therefore, the maximum contribution to a SEP for 2019 is $56,000 ($280,000 maximum compensation x 25%, limited to $56,000).

Contributing $1,500 to his retirement fund at the end of each year beginning the year he turns age 18 and ending in the year he turns age 50 (inclusive), with an average annual return of 12%, how much does Juan have in his retirement account after his last contribution to use toward a possible early retirement?

$575,281

Baldwin is the president of ZZZ Best Carpet Cleaners. Baldwin has a home with 18,000 square feet of floor space. Every week, Baldwin has the cleaning crew of ZZZ Best Carpet Cleaners clean his carpets. He theorizes that this is good training for the crew. Baldwin's CPA tells him that he can avoid the inclusion of the value of the services in his gross income if he offers to have any employees carpet cleaned at a reduction from retail. Baldwin establishes such a plan and allows a 50% reduction for all employees including himself. This year, the total retail value of the cleaning at Baldwin's house was $20,000, of which he paid $10,000. How much, if any, must Baldwin include in gross income resulting from the discount?

$6,000. Rationale $6,000 [$10,000 - ($20,000 x 0.20)]. The maximum discount excludable from an employee's gross income is 20%. Therefore, Baldwin must include an amount in his gross income equal to the total discount he received ($10,000) less the maximum excludable discount ($4,000) or $6,000.

Robin and Robbie, both age 45, are married and filed a joint return for 2019. Robbie earned a salary of $100,000 in 2019 and is covered by his employer's 401(k) plan. Robbie and Robin earned interest of $30,000 in 2019 from a joint savings account. Robin is not employed, and the couple had no other income. On April 15, 2020, Robbie contributed $6,000 to an IRA for himself and $6,000 to an IRA for Robin. The maximum allowable IRA deduction on the 2019 joint return is:

$6,000. Rationale The ability to deduct the IRA contribution depends on the individual's income and whether the individual has a qualified plan. Based on the information provided in the problem, Robin and Robbie have an AGI of $130,000 ($100,000 salary + $30,000 interest income). Since Robbie has a qualified plan, they cannot deduct the contribution for him because his income exceeds the AGI phaseout of $103,000 - $123,000 for 2019. Robin, on the other hand, can deduct her contribution because she does not have a qualified plan and their joint income is less than the $193,000 to $203,000 phaseout. Therefore, Robin's deduction is $6,000. She can use Robbie's earned income as her own.

Robin and Robbie, both age 45, are married and filed a joint return for 2019. Robbie earned a salary of $100,000 in 2019 and is covered by his employer's 401(k) plan. Robbie and Robin earned interest of $30,000 in 2019 from a joint savings account. Robin is not employed, and the couple had no other income. On April 15, 2020, Robbie contributed $6,000 to an IRA for himself and $6,000 to an IRA for Robin. The maximum allowable IRA deduction on the 2019 joint return is:

$6,000. Rationale The ability to deduct the IRA contribution depends on the individual's income and whether the individual has a qualified plan. Based on the information provided in the problem, Robin and Robbie have an AGI of $130,000 ($100,000 salary + $30,000 interest income). Since Robbie has a qualified plan, they cannot deduct the contribution for him because his income exceeds the AGI phaseout of $103,000 - $123,000 for 2019. Robin, on the other hand, can deduct her contribution because she does not have a qualified plan and their joint income is less than the $193,000 to $203,000 phaseout. Therefore, Robin's deduction is $6,000. She can use Robbie's earned income as her own.

Jack and Jill, both age 43, are married, made $20,000 each, and file a joint tax return. Jill has made a $6,000 contribution to her Traditional IRA account and has made a contribution of $2,000 to a Coverdell Education Savings Account for 2019. What is the most that can be contributed to a Roth IRA for Jack for 2019?

$6,000. Rationale The maximum combined contribution to traditional and Roth IRAs is $6,000 per person (who has not attained age 50) for 2019. Therefore, Jack and Jill would have a total of $12,000 to allocate between traditional and Roth IRAs. Jill has already contributed the maximum amount; however, Jack could still contribute $6,000 for himself. The Coverdell Education Savings Account (formerly known as an Education IRA) is not included in the $6,000 limit.

Frank, divorced and age 56, received wages of $45,000 in 2018. In addition, he received $1,800 in earnings from a part-time job. Frank is not covered by a qualified plan. What was the maximum deductible IRA contribution that Frank could have made for 2018?

$6,500

Eric moved from Houston to New Orleans in 2019. His expenses for the move included $400 for truck rental, $100 for lodging, and $200 of pre-move house-hunting expenses. If Eric's employer reimbursed him $600, how much of the reimbursement is included in his gross income for 2018?

$600. Rationale TCJA 2017 suspends the exclusion from gross income for moving expenses reimbursement for years 2018-2025, except for armed service members. Thus, $600 is included in gross income for Eric. Prior to 2018 and after 2025: Eric has $500 of qualifying expenditures. $400 for the truck rental and $100 for the lodging. Pre-move house-hunting expenses are not qualifying moving expenses. Any reimbursement in excess of $500 is included in his gross income. Thus, $100 ($600 - $500) is included in his gross income.

Delores, age 62, single, and retired, receives a defined benefit pension annuity of $1,200 per month from Bertancinni Corporation. She is currently working part time for Deanna's Interior Design and will be paid $18,000 this year (2019). Deanna's Interior has a 401(k) plan, but Delores has made no contribution to the plan and neither will Deanna this year. Can Delores contribute to a traditional IRA or a Roth IRA for the year and what is the maximum contribution for 2019?

$7,000 to a traditional IRA or $7,000 to a Roth IRA. Rationale Delores has earned income and is over age 50. She is not an active participant in a retirement plan and even if she were, her AGI is below the income limits.

Amy, divorced and age 55, received taxable alimony of $50,000 in 2019. In addition, she received $1,800 in earnings from a part-time job. Amy is not covered by a qualified plan. What was the maximum deductible IRA contribution that Amy could have made for 2019?

$7,000. Rationale The deductible IRA contribution limit is $6,000 for 2019. The additional catch-up amount, for over age 50, is $1,000 for 2019. Alimony counts as earned income for IRA purposes for any divorce or separation agreement executed prior to 2019 (2017 TCJA). She is not covered by a qualified plan and therefore is not subject to AGI phase-outs. Therefore the total is $7,000 for 2019.

Beth Ann, age 55, is an attorney who is self employed. Her assistant, Buffy, has worked with her for five years. Beth Ann is establishing a SEP for this year and is willing to make a contribution of 10 percent of Buffy's salary to the SEP. If Beth Ann earns $90,000 after paying Buffy, her expenses, and the contribution to Buffy's SEP, what is the most that she can contribute to the SEP on her behalf for 2019?

$7,604. Rationale $90,000 Schedule C Net Income $6,358 Less 1/2 Self Employment Tax $90,000 x 92.35% x 15.3% x 1/2 $83,642 x 9.09% (10%/1.10) $7,604 SEP Contribution Limit Beth Ann is limited to the same percentage contribution as her employer. However, because she is self employed, the 10% is based on earned income. The short-cut is to use 9.09% (10% / 1.10).

Jackie and Andrew, both age 40, have $80,000 of combined retirement assets. They both expect to retire at the age of 65 with a joint life expectancy of 30 years. They expect to earn 10%, compounded annually, on the assets within their retirement accounts before retirement and 8%, compounded annually, during their retirement. If they did not make any additional contributions to their account and they receive a fixed annual annuity benefit for life, what is the annual benefit they will receive during retirement?

$71,290

Greg is employed by a large corporation with 400 employees. The corporation provides its employees with a no-cost gym membership at a local YMCA. The cost of the membership is $60 per month, which is completely paid for by Greg's employer. How much must Greg include in his yearly gross income related to this no-cost fringe benefit?

$720. Rationale In order for the value of the athletic facilities to be excluded from an employee's gross income, the athletic facilities must be located on the employer's business premises and must be for the exclusive use of the employees and their dependents. Greg's membership is at a public YMCA and is therefore taxable. 12 x $60 = $720.

Marleen, who is 37 years old, is an employee of Zcrypt, Inc. (Zcrypt). Zcrypt sponsors a SEP IRA and would like to contribute the maximum amount to Marleen's account for the plan year. If Marleen earns $32,000 per year, what is the maximum contribution Zcrypt can make on her behalf to the SEP IRA?

$8,000

Isse Peking is the manager of Airline Highway Motel. Isse lives in Unit 12. He was given the option to live at the motel if he would also look after the night auditing (the value of his reviews is $400 per month) responsibilities. The value of the motel unit on a monthly basis is $800, but Unit 12 rents on a daily basis for $100 per day. How much, if any, does Isse have to include in his gross income for living on the premises of his employer?

$800 per month. Rationale Isse is not required by the employer to live on the premises and therefore must include the value of the lodging in his gross income.

Antoine immigrated from Italy last century, became a citizen and has worked the better part of his life in the United States, for which he is truly thankful. His full retirement age (normal retirement age) for Social Security benefits is age 66, but after a hard life working he wants to retire at age 63 and travel throughout America and back to his homeland. If his benefit at age 66 is $1,000 per month, how much will he receive in Social Security retirement benefits if he begins receiving benefits at age 63?

$800.00. Rationale The benefit reduction for early retirement is 5/9ths of 1% for the first 36 months. If Antoine retires 3 years early at age 63, then his retirement benefit will be reduced by 20% to $800 per month.

Roy and Barbara are near retirement. They have a joint life expectancy of 25 years in retirement. Barbara anticipates their annual spending in retirement will need to increase each year at the rate of inflation, which they assume is 4%. Based on the assumption that their first year retirement spending will be $85,000, of which they have $37,500 available from other sources (which will also increase with inflation every year at the same rate) and an annual nominal rate of return of 6.5%, calculate the total amount that needs to be in place when Roy and Barbara begin their retirement. Don't consider taxes in your calculation. Correct Answer

$906,131

Dan owns and operates Schoepf's Sales Solutions (3S), a sole proprietorship. 3S sponsors a profit-sharing plan. Dan had net income of $250,000 and paid self-employment taxes of $22,000 (assumed) during the year. Assuming Dan is over the age of 50, what is the maximum amount that Dan can contribute to the profit sharing plan on his behalf for 2018?

. $47,800. $250,000 Net Income ($11,000) Less 1/2 SE Tax $239,000 Net SE Income x 0.20 0.25/1.25 $47,800 The fact that Dan is over the age of 50 is not relevant. Catch up contributions are not made by employers, only by employees to CODAs.

Employees generally contribute to which of the following plans? 401(k) plans. Thrift plans. Cash balance pension plans. Defined benefit pension plans.

1 and 2

Which of the following is false as to adoption assistance programs? 1. The amount paid is excluded from the employee's income regardless of the employee's income level. 2. The unlimited amount paid is excluded from the employee's income with a phaseout of AGI at certain levels. 3. Up to $14,080 for 2019 is excluded from the employee's income with a phaseout of AGI at certain levels.

1 and 2. Rationale Statements 1 and 2 are false. Statement 1 does not limit the expense paid nor does it acknowledge the phaseout of $211,160 through $251,160 for 2019. Statement 2 does not reveal that there is a limit on the expense for adoption assistance programs.

Investment portfolio risk is generally borne by the participant/employee in all of the listed qualified plans, except: 1. Defined benefit pension plan. 2. Cash balance pension plan. 3. 401(k) plan. 4. Profit sharing plan.

1 and 2. Rationale In defined benefit and cash balance pension plans, the employer bears the investment risk.

A SEP is not a qualified plan and is not subject to all of the qualified plan rules. However, it is subject to many of the same rules. Which of the following are true statements? 1. SEPs and qualified plans have the same funding deadlines. 2. The contribution limit for SEPs and qualified plans (defined contribution) is $56,000 for the year 2019. 3. SEPs and qualified plans have the same ERISA protection from creditors. 4. SEPs and qualified plans have different nondiscriminatory and top-heavy rules.

1 and 2. Rationale SEPs and qualified plans can be funded as late as the due date of the return plus extensions. The maximum contribution for an individual to a SEP is $56,000 for 2019 ($280,000 maximum compensation x 25%, limited to $56,000). Thus, statements 1 and 2 are correct. Qualified plans are protected under ERISA. IRAs and SEPs do not share this protection. Both types of plans have the same nondiscriminatory and top-heavy rules.

Which of the following are permitted investments in a 403(b) TSA (TDA) plan? 1. An annuity contract from an insurance company. 2. An international gold stock mutual fund. 3. A self-directed brokerage account consisting solely of U.S. stocks, bonds and mutual funds.

1 and 2. Rationale TSA (TDA) funds can only invest in annuity contracts (Statement 1) and mutual funds (Statement 2). No self-directed brokerage accounts are permitted.

Employees generally contribute to which of the following plans? 1. 401(k) plans. 2. Thrift plans. 3. Cash balance pension plans. 4. Defined benefit pension plans.

1 and 2. Rationale Cash balance pension plans and defined benefit pension plans are almost always exclusively funded by the employer (noncontributory). 401(k) plans and thrift plans allow employee contributions.

Which of the following are benefits of converting assets in a qualified plan to a Roth account through an in-plan Roth rollover? 1. The conversion may result in a reduction in income tax in future years. 2. The conversion will result in increasing after-tax deferred assets and reducing the gross estate. 3. The conversion will eliminate the need for minimum distributions during the life of the participant.

1 and 2. Rationale While there are no guarantees, the conversion may result in a reduction in tax in future years since all future income in the account will escape taxation. The conversion does result in increasing after-tax deferred assets and reducing the gross estate. However, because the funds are in a qualified plan, they will have to be distributed to comply with the minimum distribution rules or be rolled over to a Roth IRA.

When offering a qualified profit sharing plan, employers have choices when it comes to vesting schedules. What are slowest vesting schedules that employers can select for vesting employer contributions? 1. Five years cliff vesting 2. Three years cliff vesting 3. Two to six years graded vesting 4. Three to seven years graded vesting

1 and 3

Which of the following is/are correct regarding SIMPLE plans? 1. A SIMPLE plan does not require annual testing. 2. A SIMPLE IRA must follow a 3-year cliff vesting schedule if the plan is top- heavy. 3. A 25% early withdrawal penalty may apply to distributions taken within the first two years of participation in a SIMPLE plan. 4. The maximum elective deferral contribution to a SIMPLE 401(k) plan is $19,000 for 2019 and $25,000 for 2019 for an employee who has attained the age of 50.

1 and 3. Rationale Statement 1 is correct. Statement 2 is incorrect. A SIMPLE plan is not subject to vesting rules, and contributions are always a 100% vested. Statement 3 is correct. The early withdrawal penalty is 25% for distributions taken within the first two years of participation. Statement 4 is incorrect. The maximum deferral to a SIMPLE plan is $13,000 for 2019. Employees who have attained age 50 by the end of the tax year will also be eligible for a catch-up adjustment ($3,000 for 2019).

Which of the following are correct? 1. SIMPLEs provide incentives to small employers to adopt retirement plans for employees with less administrative costs and fewer set-up procedures than qualified plans. 2. SIMPLE IRAs can permit loans to employees. 3. SIMPLE IRAs require the employer either to match the employee contributions of those who participate or to provide nonelective contributions to all eligible employees.

1 and 3. Rationale Statement 2 is false. Loans are not permitted from any IRA. Statements 1 and 2 are correct.

ABC has an Employee Stock Purchase Plan (ESPP). Which statement(s) regarding an ESPP is/are correct? 1. The purchase price of the stock may be as low as 85% of the stock value. 2. When an employee sells ESPP stock at a gain in a qualifying disposition, all of the gain is capital gain. 3. There is an annual limit of $25,000 per employee for ESPPs.

1 and 3. Rationale Statement 2 is incorrect because only the gain in excess of the W-2 income will be capital gain. Statements 1 and 3 are correct.

Which of the following statements is/are correct regarding 403(b) plans? 1. 403(b) plans are eligible for rollover treatment to IRAs, qualified plans, and other 403(b) plans. 2. Investments in stocks, bonds, and money markets are available. 3. Assets in a 403(b) plan are generally 100% vested.

1 and 3. Rationale Statements 1 and 3 are correct. Statement 2 is incorrect because investments are limited to insurance annuities and mutual funds.

Generally, older age entrants are favored in which of the following plans? 1. Defined benefit pension plans. 2. Cash balance pension plans. 3. Target benefit pension plans. 4. Money purchase pension plans.

1 and 3. Rationale Cash balance and money purchase pension plans generally favor younger age entrants. While defined benefit and target benefit pension plans favor older age entrants with less time to accumulate and require higher funding levels.

The early distribution penalty of 10 percent does not apply to qualified plan distributions: 1. Made after attainment of the age of 55 and separation from service. 2. Made for the purpose of paying qualified higher education costs. 3. Paid to a designated beneficiary after the death of the account owner who had not begun receiving minimum distributions.

1 and 3. Rationale Statement 2 is an exception for distributions from IRAs, not qualified plans. Statements 1 and 3 are exceptions to the 10% penalty for qualified plan distributions

Which of the following are correct? 1. SIMPLEs provide incentives to small employers to adopt retirement plans for employees with less administrative costs and fewer set-up procedures than qualified plans. 2. SIMPLE IRAs can permit loans to employees. 3. SIMPLE IRAs require the employer either to match the employee contributions of those who participate or to provide nonelective contributions to all eligible employees.

1 and 3. Rationale Statement 2 is false. Loans are not permitted from any IRA. Statements 1 and 2 are correct.

Which of the employee fringe benefits listed below, if provided by the employer, are both deductible by the employer and not included in an employee's gross income after 2017? 1. Business periodical subscriptions. 2. Season tickets to professional football games. 3. Parking provided near its business (employer pays $90 per month per employee). 4. The use of an on-premises athletic facilities (value of $180 per month).

1 and 4. Rationale Season tickets to professional football games are includible in the gross income of the employee receiving the tickets. Periodicals and athletic facilities are both deductible and not included in gross income. Effective with TCJA 2017, the other choices are not deductible by the employer. Qualified transportation fringe benefits are not deductible after 2017 as a result of TCJA 2017.

Which of the following statements is/are correct regarding TSAs and 457(b) deferred compensation plans? 1. Both plans require contracts between an employer and an employee. 2. Participation in either a TSA or a 457 plan will cause an individual to be considered an "active participant for purposes of phasing out the deductibility of Traditional IRA contributions. 3. Both plans allow 10-year forward averaging tax treatment for lump-sum distributions. 4. Both plans must meet minimum distribution requirements that apply to qualified plans.

1 and 4. Rationale Statements 1 and 4 are correct. Statement 2 is incorrect because a 457 plan is a deferred compensation arrangement that will not cause a participant to be considered an "active participant." Statement 3 is incorrect because 10-year forward averaging is not permitted from either plan, since neither is a qualified plan.

Which of the following distributions from a qualified plan would not be subject to the 10% early withdrawal penalty, assuming the participant has not attained age 59½? 1. A distribution made to a spouse under a Qualified Domestic Relations Order (QDRO). 2. A distribution from a qualified plan used to pay the private health insurance premiums of a current employee of Clinical Trials Company. 3. A distribution to pay for costs of higher education. 4. A distribution made immediately after separation from service at age 57.

1 and 4. Rationale Statement 2 is incorrect for two reasons. The exception to the 10 percent early withdrawal penalty for health insurance premiums is only applicable to unemployed individuals. In addition, this exception is only available for distributions from IRAs, not qualified plans. Statement 3 is incorrect because the exception to the 10 percent penalty for higher education expenses only applies to distributions from IRAs, not qualified plans.

Which of the following statements is/are correct regarding TSAs and 457(b) deferred compensation plans? 1. Both plans require contracts between an employer and an employee. 2. Participation in either a TSA or a 457 plan will cause an individual to be considered an "active participant for purposes of phasing out the deductibility of Traditional IRA contributions. 3. Both plans allow 10-year forward averaging tax treatment for lump-sum distributions. 4. Both plans must meet minimum distribution requirements that apply to qualified plans.

1 and 4. Rationale Statements 1 and 4 are correct. Statement 2 is incorrect because a 457 plan is a deferred compensation arrangement that will not cause a participant to be considered an "active participant." Statement 3 is incorrect because 10-year forward averaging is not permitted from either plan, since neither is a qualified plan.

Which of the following taxpayers is/are eligible to make contributions to a Roth IRA? Jack and Catelyn file jointly and have an AGI of $145,000 Bryce and Madison file separately and each have an AGI of $40,000 Carlos files as a single person and has an AGI of $175,000

1 only

Which of the following types of 457 plans permit employees to defer recognition of income without any risk of forfeiture? 1. Public 457(b) plans. 2. 457(f ) plans. 3. Private 457(b) plans.

1 only. Rationale Assets in 457(f ) plans and private 457(b) plans must be subject to a substantial risk of forfeiture.

All of the following statements is/are correct regarding tax-sheltered annuities (403(b) plans) except? 1. The non-age-based catch-up provision is available to employees of all 501(c)(3) organization employers that sponsor a TSA. 2. Active employees who take withdrawals from TSAs prior to 59½ are subject to a 10% penalty tax. 3. TSAs are available to all employees of 501(c)(3) organizations who adopt such a plan. 4. If an employee has had at least 15 years of service with an eligible employer, an additional catch-up contribution may be allowed.

1 only. Rationale Statement 1 is incorrect. The catch-up provision requires specified service and the correct kind of employer. Statements 2, 3, and 4 are correct.

Which of the following types of 457 plans permit employees to defer recognition of income without any risk of forfeiture? 1. Public 457(b) plans. 2. 457(f ) plans. 3. Private 457(b) plans.

1 only. Rationale Assets in 457(f ) plans and private 457(b) plans must be subject to a substantial risk of forfeiture.

Which of these account types have "Roth" options? 403(b) 401(k) 457 Plan SEP-IRA

1, 2 and 3

Which of the following people would be considered a highly compensated employee for 2018? Amy a 2% owner whose salary last year was $165,000. Red, a 6% owner whose salary was $23,500 for the last five years. Reese, an officer, who earned $115,000 last year and is the sixth highest paid employee of 96 employees. Hank, a 0.5% owner who earned $127,000 last year and is in the top 20% of paid employees.

1, 2 and 4

Which of the following premiums for health insurance provided by an employer is/are excludable from income tax by the employee? Premiums for the employee if currently employed. Premiums for the employee's spouse. Premiums for the employee's dependents other than a spouse. Premiums for the employee if retired.

1, 2, 3, and 4.

Which of the following premiums for health insurance provided by an employer is/are excludable from income tax by the employee? 1. Premiums for the employee if currently employed. 2. Premiums for the employee's spouse. 3. Premiums for the employee's dependents other than a spouse. 4. Premiums for the employee if retired.

1, 2, 3, and 4. Rationale All premiums (Statements 1-4) are excludable.

Qualified retirement plans that permit the employer unlimited investment in sponsor company stock are: 1. 401(k) plans. 2. Stock bonus plans. 3. Profit sharing plans. 4. ESOPs.

1, 2, 3, and 4. Rationale All of the listed plans permit 100% stock in the plans. The 401(k) plan is organized as a profit sharing or stock bonus plan.

Which of the following is/are advantages of cafeteria plans? 1. Cafeteria plans help to give employees an appreciation of the value of their benefit package. 2. The flexibility of a cafeteria benefit package helps to meet varied employee needs. 3. Cafeteria plans can help control employer costs for the benefit package because the cost of benefits that employees do not need is minimized. 4. Cafeteria plans are less complex and less expensive to design and administer than general group benefit plans.

1, 2, and 3. Rationale Cafeteria plans are more complex and expensive to design and administer. All of the other statements are true.

Which of the following people can make a deductible contribution to a traditional IRA for 2019? Person AGI Covered by Qualified Plan Marital Status 1. Dianne $100,000 Yes Married 2. Joy $50,000 Yes Single 3. Kim $280,000 No Married 4. Loretta $79,000 Yes Single

1, 2, and 3. Rationale All but Loretta may deduct a contribution made to a traditional IRA. Dianne and Joy are below the phaseout range and Kim is not covered by a qualified plan so there is no income limit. Loretta is single and covered by a plan and her AGI is above the top end of the phaseout for singles ($64,000 - $74,000) for 2019.

Which of the following characteristics accurately describes a 403(b) plan? 1. A self-reliant employee elective deferral plan. 2. The retirement benefit is dependent on the investment results. 3. The plan generally permits loans.

1, 2, and 3. Rationale All of the features described are common to 403(b) plans.

Which of the following plans permit employers to match employee elective deferral contributions or make non-elective contributions? 1. 457(b) plan. 2. 401(k) plan. 3. 403(b) plan.

1, 2, and 3. Rationale All three plans permit employer matching and non-elective contributions. The 457 employer contribution goes against the $18,000 limit, whereas employer contributions do not go against the $18,000 limit for 401(k) and 403(b) plans.

Which of the following statements is/are correct regarding the early distribution 10 percent penalty tax from a qualified plan? 1. Retirement at age 55 or older exempts the distributions from the early withdrawal penalty tax. 2. Distributions used to pay medical expenses in excess of the 10% of AGI for a tax filer who itemizes are exempt from the early withdrawal penalty. 3. Distributions that are part of a series of equal periodic payments paid over the life or life expectancy of the participant are exempt from the early withdrawal penalty.

1, 2, and 3. Rationale Statements 1, 2, and 3 are correct. The TCJA 2017 reduced the 10% AGI limit to 7.5% of AGI for medical deductions for 2017 and 2018.

Which of the following people would be considered a highly compensated employee for 2018? 1. Kim, a 1% owner whose salary last year was $150,000 2. Rita, a 6% owner whose salary was $42,000 last year. 3. Robin, an officer, who earned $105,000 last year and is the 29th highest paid employee out of 96 employees 4. Helen, who earned $132,000 last year and is in the top 20 percent of paid employees.

1, 2, and 4

Which of the following is true regarding employer contributions to secular trusts for employee-participants of a nonqualified deferred compensation agreement? 1. Participants have security against an employer's unwillingness to pay at termination. 2. Participants have security against an employer's bankruptcy. 3. Secular trusts provide tax deferral for employees until distribution. 4. Secular trusts provide employers with a current income tax deduction.

1, 2, and 4. Rationale Secular trusts are similar to rabbi trusts except that participants do not have a substantial risk of forfeiture and thus, do not provide the employee with tax deferral. Secular trusts provide the employer with a current income tax deduction for contributions. Secular trusts protect the participant from employer unwillingness to pay because they are funded and they protect from bankruptcy because there is no risk of forfeiture.

Which of the following qualified plans require mandatory funding? 1. Defined benefit pension plans. 2. 401(k) plans with an employer match organized as a profit sharing plan. 3. Cash balance pension plans. 4. Money purchase pension plans.

1, 3, and 4. Rationale 401(k) plans do not require mandatory funding. The other three require mandatory funding.

Taylor, age 25, works for Swim America. Swim America adopted a SIMPLE plan six months ago. Taylor made an elective deferral contribution to the plan of $8,000, and Swim America made a matching contribution of $2,400. Which of the following statements is/are correct? 1. Taylor can withdraw his entire account balance without terminating employment. 2. Taylor can roll his SIMPLE IRA into his Traditional IRA. 3. Taylor will be subject to ordinary income taxes on withdrawals from the SIMPLE. 4. Taylor may be subject to a 25% early withdrawal penalty on amounts withdrawn from the SIMPLE.

1, 3, and 4. Rationale Statement 1 is correct. A participant can take a distribution from a SIMPLE at any time without separating from service. SIMPLEs must provide 100% immediate vesting of employer contributions. The entire balance is available for withdrawal. Statement 2 is incorrect. A SIMPLE IRA cannot be rolled in to a traditional IRA until the participant has been in the SIMPLE IRA for two years. Tyler has only been in the SIMPLE for six months. Statement 3 is correct. The entire withdrawal will be subject to ordinary income tax in the year of withdrawal. Statement 4 is correct be cause the early withdrawal penalty for a SIMPLE is 25% for withdrawals occurring within the first two years of participation.

GreatGrains Corporation just put a safe harbor 401(k) profit sharing plan into place. Which of the following tests may be avoided with the safe harbor provisions? Top heavy rules ADP test ACP test Nondiscrimination tests including coverage rules

1,2 and 3

Qualified retirement plans that permit the employer unlimited investment in sponsor company stock are: 401(k) plans. Stock bonus plans. Profit sharing plans. ESOPs.

1,2,3, and 4

What are three benefits of a Roth Conversion?

1. Tax Arbitrage (ability to pay a lower income tax right now on the funds if you expect to be in a higher brakes in the future.) 2. Easy Access to the funds (after waiting the 5 year participation period) 3. No Required Minimum Distributions 4. Tax diversification (having pre tax and after tax dollars)

Your client is retiring at age 57 from a local private employer with a balance of $300,000 in her 401(k) plan. She asks you whether she should roll over the balance into an IRA. What are three potential good reasons you could cite that would support keeping your funds in the 401(k) with the employer.

1. Tax Arbitrage: Keeping it in the 401k will allow the money to grow without tax until she is in a lower income bracket where she will pay less income tax on the 401k funds than moving it now when she is still in higher income bracket for the taxable year. 3. The earnings will continue to grow on a tax deferred basis in the 401(K) but if she rolls the funds over she will not be able to reverse into a 401(k) later.

Kenny holds two jobs - a full-time job with R Corporation and a part-time job with Z Corporation. Kenny uses his car to drive to work. The mileage is as follows: from Kenny's home to R is 70 miles; from R to Z is 10 miles; and from Z to Kenny's home is 70 miles. Kenny's deductible mileage for each work day is:

10 miles. Rationale The deduction is based on the distance between jobs. There is no deduction for commuting expenses (mileage) to or from a taxpayer's primary place of employment.

The target benefit pension plan and the money purchase pension plan provide some employee/participant investment diversification protections by limiting the investment amount in employer stock to less than or equal to:

10%. Rationale Defined benefit, cash balance, target benenit, and money purchase pension plans limit contributions of company stock to 10%.

What is the maximum number of employees that a company with a health plan can have and not be subject to the COBRA rules?

19 A company with less than 20 employees is not subject to COBRA even if they have a health plan.

If a qualified plan has been designed using normal eligibility requirements, which combination of the following would mean the employee could participate in the plan? 1. 18 years of age 2. 21 years of age 3. Completing 1 year of service with at least 1,000 hours worked per year 4. Completing 3 years of service averaging 500 hours of work a year

2 and 3

Rick has an 18% nonqualified deferred compensation plan that is funded annually by his employer. Payments are made to a separate trustee of a secular trust who was selected by Rick and his employer. The employer contributions are discontinued at Rick's death, disability, or employment termination. When Rick retires or terminates employment, he will receive the proceeds from the trust. Which of the following is/are correct regarding the deferred compensation plan? 1. The contributions are not currently taxable to Rick because they are subject to a substantial risk of forfeiture. 2. The contributions to the plan are currently subject to payroll taxes. 3. The employer can deduct the contributions to the plan at the time of the contribution.

2 and 3. Rationale Because this arrangement is a secular trust, there is no substantial risk of forfeiture to Rick. Thus, Statement 1 is false. Because the trust is not subject to the general creditors of the employer, this is straight compensation. Rick must treat the payments as constructively received, and the employer may deduct the payments as compensation immediately. The payments are subject to payroll tax since the compensation is currently earned.

The early distribution penalty of 10 percent does not apply to IRA distributions: 1. Made after attainment of the age of 55 and separated from service. 2. Made for the purpose of paying qualified higher education costs. 3. Paid to a designated beneficiary after the death of the account owner who had not begun receiving minimum distributions.

2 and 3. Rationale The first statement is incorrect because it is an exception to the 10% penalty for qualified plan distributions, not from IRAs. The second and third statements are correct exceptions for IRAs.

Which of the following is/are elements of an effective waiver for a pre-retirement survivor annuity? 1. Both spouses must sign the waiver. 2. The waiver must be notarized or signed by a plan official. 3. The waiver must indicate that the person(s) waiving understand the consequences of the waiver.

2 and 3. Rationale Only the nonparticipant spouse must sign the waiver.

David took a lump-sum distribution from his employer's qualified plan at age 56 when he terminated his service. He rolled over his distribution using a direct rollover to an IRA. Assuming David has met 10-year forward averaging requirements, which of the following is/are correct regarding tax treatment of the transaction? 1. If at age 59 he distributes the IRA, he benefits from 10-year forward averaging. 2. If he rolls the entire IRA to a new employer's qualified plan, he may be eligible for forward averaging treatment in the future. 3. If he rolls over a portion of the IRA to a new employer's qualified plan, he may preserve any eligibility for forward averaging on that portion that was rolled over. 4. If David immediately withdraws the entire amount from his IRA, he may benefit from 10-year forward averaging.

2 and 3. Rationale Statement 1 is incorrect because 10-year forward averaging is only permitted with qualified plans, not IRAs. Statements 2 and 3 are correct. His new employer's qualified plan may or may not allow him to roll previous distributions into it. Statement 4 is incorrect because 10-year forward averaging is only permitted coming from qualified plans, not distributions from IRAs.

$6,000. Rationale The ability to deduct the IRA contribution depends on the individual's income and whether the individual has a qualified plan. Based on the information provided in the problem, Robin and Robbie have an AGI of $130,000 ($100,000 salary + $30,000 interest income). Since Robbie has a qualified plan, they cannot deduct the contribution for him because his income exceeds the AGI phaseout of $103,000 - $123,000 for 2019. Robin, on the other hand, can deduct her contribution because she does not have a qualified plan and their joint income is less than the $193,000 to $203,000 phaseout. Therefore, Robin's deduction is $6,000. She can use Robbie's earned income as her own.

2 and 3. Rationale The first statement is incorrect because it is an exception to the 10% penalty for qualified plan distributions, not from IRAs. The second and third statements are correct exceptions for IRAs.

The chief financial officer of a new company wishes to install a retirement plan in which the employer would be making decisions on how pension funds are invested rather than participants. Identify the plan(s) below which must meet this requirement. Profit sharing plan Cash balance plan Target benefit plan Defined-benefit plan

2 and 4 only

Which of these scenarios have required minimum distributions: George is age 72 and participates in a 401(k) through his employer. Katie, age 53, inherited a Roth IRA from her mother two years ago. Fred separated from service from his non-profit employer two years ago, when he was 65 years old. He has a 403(b) balance from that service and he has taken qualified distributions from the plan. Devon, age 68, transferred the balance of his deceased spouse's IRA into his own IRA last year. She had been taking required distributions before her death.

2 only

Which of these statement are/is true? Roth IRA earnings can be withdrawn at any time, at any age free of penalty or tax. Each person under age 70 ½ with sufficient earned income may contribute to a traditional IRA Indirect rollovers from a 401(k) profit sharing plan to an IRA will not have tax withholding as long as participant deposits the check from the plan into the IRA within 60 days.

2 only

A business valued at $3,000,000 has 3 partners. Each of the 3 partners buys a $500,000 life insurance policy for purposes of a buy/sell agreement on each of the other partners. Which of the following is/are true? 1. This is an example of an entity purchase plan. 2. This is an example of a cross purchase plan. 3. The buy/sell agreement is under funded

2 only. Rationale This is a cross-purchase life insurance plan. Each person has a one-third interest. Therefore, when the first partner dies, the other two partners will each need to pay $500,000 for a total of $1,000,000 (1/3 of $3,000,000). Thus, the buy/sell agreement is not underfunded.

Betty Sue, age 75, is a widow with no close relatives. She is very ill, unable to walk, and confined to a custodial nursing home. Which of the following programs is likely to pay benefits towards the cost of the nursing home? 1. Medicare may pay for up to 80 additional days of care after a 20-day deductible. 2. Medicaid may pay if the client has income and assets below state-mandated thresholds.

2 only. Rationale Statement 1 is incorrect because Medicare covers all costs for the first 20 days of skilled nursing home care and covers the next 80 days with a deductible.

A distress termination of a qualified retirement plan occurs when: 1. The PBGC initiates a termination because the plan was determined to be unable to pay benefits from the plan. 2. An employer is in financial difficulty and is unable to continue with the plan financially. Generally, this occurs when the company has filed for bankruptcy, either Chapter 7 liquidation or Chapter 11 reorganization. 3. The employer has sufficient assets to pay all benefits vested at the time, but is distressed about it. 4. When the PBGC notifies the employer that it wishes to change the plan due to the increasing unfunded risk.

2 only. Rationale Statement 2 is the definition of a distress termination. Statement 3 describes a standard termination. Statement 1 describes an involuntary termination. Statement 4 is simply false.

Which of the following statements is/are correct regarding SEP contributions made by an employer? 1. Contributions are subject to FICA and FUTA. 2. Contributions are currently excludable from employee-participant's gross income. 3. Contributions are capped at $19,000 for 2019.

2 only. Rationale Statement 2 is the only correct response. Statements 1 and 3 are incorrect. Employer contributions to a SEP are not subject to FICA and FUTA. The 401(k) elective deferral limit and the SARSEP deductible limits are $19,000 for 2019. The SEP limit is 25% of covered compensation up to $56,000 for 2019. Note: The maximum compensation that may be taken into account in 2019 for purposes of SEP contributions is $280,000. Therefore, the maximum amount that can be contributed to a SEP in 2019 is $56,000 (25% x $280,000, limited to $56,000).

Which of the following statements is/are correct regarding SEP contributions made by an employer? 1. Contributions are subject to FICA and FUTA. 2. Contributions are currently excludable from employee-participant's gross income. 3. Contributions are capped at $19,000 for 2019.

2 only. Rationale Statement 2 is the only correct response. Statements 1 and 3 are incorrect. Employer contributions to a SEP are not subject to FICA and FUTA. The 401(k) elective deferral limit and the SARSEP deductible limits are $19,000 for 2019. The SEP limit is 25% of covered compensation up to $56,000 for 2019. Note: The maximum compensation that may be taken into account in 2019 for purposes of SEP contributions is $280,000. Therefore, the maximum amount that can be contributed to a SEP in 2019 is $56,000 (25% x $280,000, limited to $56,000).

A client walks into your office excited about her access to a 457(b) plan through a private non-profit hospital. In weighing the pros and cons of contributing to such as plan, which of the below statements are true? If she's already maxing out her 403(b) plan contributions, then she won't be able to contribute to the 457(b) plan. The 457(b) plan could put her savings a risk in case the non-profit were declared insolvent. The 457(b) plan has limited rollover options once she leaves the hospital. Like public 457(b) plans, private plans have a catch-up provision in the final three years before retirement age.

2, 3 and 4

What is the first year in which a single taxpayer, who turned age 54 in March 2016, could receive a qualified distribution of earnings from a Roth IRA if he made his first ever $3,500 contribution to a Roth IRA on April 1, 2017, for the tax year 2016?

2021

What is the first year in which a single taxpayer, age 54 in 2019, could receive a qualified distribution from a Roth IRA if he made his first $3,500 contribution to the Roth IRA on April 1, 2020, for the tax year 2019?

2024. Rationale A qualified distribution can only occur after a five-year period has occurred and is made on or after the date on which the owner attains age 59½, made to a beneficiary or the estate of the owner on or after the date of the owner's death, attributable to the owner's being disabled, or for a first-time home purchase. The five-year period begins at the beginning of the taxable year of the initial contribution to a Roth IRA. The five-year period ends on the last day of the individual's fifth consecutive taxable year beginning with the taxable year described in the preceding sentence. Therefore, the first year in which a qualified distribution could occur is 2024.

Investment portfolio risk is generally borne by the employer in all of the listed qualified plans, except: Defined benefit pension plan Cash balance pension plan 401(k) plan Profit sharing plan

3 and 4

A self-employed client comes to you in January disclosing that they had much more income then they had anticipated. What plan(s) could you recommend be established now to save taxes from the previous year? Solo 401(k) plan 457 Plan SEP-IRA SIMPLE-IRA

3 only

Which of the following is/are false as to ESOPs? 1. An ESOP is controlled through a trust. 2. ESOPs provide corporate owners with a way to transfer ownership interests to their employees. 3. The trust of an ESOP is prohibited from borrowing money from a bank to purchase the employer stock.

3 only

Which of the following circumstances suggest the use of a cafeteria plan? 1.A cafeteria plan is appropriate when the employee mix is comprised only of older employees with families who need maximum medical and life insurance benefits. 2. A cafeteria plan is appropriate when employers want to choose the benefit package most suited to their employee's individual needs. 3. A cafeteria plan is appropriate when an employer seeks to maximize employee satisfaction with the benefit package, thereby maximizing the employer's benefit from its compensation expenditures. 4. A cafeteria plan is appropriate for a small employer who does not have much money to spend on benefits.

3 only. Rationale A cafeteria plan is appropriate when the employee mix includes young, unmarried people with minimal life insurance and medical benefits needs, as well as older employees with families who need maximum medical and life insurance benefits. A cafeteria plan is appropriate when employees want to choose the benefit package most suited to their individual needs. A cafeteria plan is also appropriate when the employer is large enough to afford the expense of such a plan.

Which of the following circumstances suggest the use of a cafeteria plan? 1.A cafeteria plan is appropriate when the employee mix is comprised only of older employees with families who need maximum medical and life insurance benefits. 2. A cafeteria plan is appropriate when employers want to choose the benefit package most suited to their employee's individual needs. 3. A cafeteria plan is appropriate when an employer seeks to maximize employee satisfaction with the benefit package, thereby maximizing the employer's benefit from its compensation expenditures. 4. A cafeteria plan is appropriate for a small employer who does not have much money to spend on benefits.

3 only. Rationale A cafeteria plan is appropriate when the employee mix includes young, unmarried people with minimal life insurance and medical benefits needs, as well as older employees with families who need maximum medical and life insurance benefits. A cafeteria plan is appropriate when employees want to choose the benefit package most suited to their individual needs. A cafeteria plan is also appropriate when the employer is large enough to afford the expense of such a plan.

What is the 4 percent approach for managing retiree income? How does inflation factor into it?

4% Approach is the rule of thumb used to save for retirement when nothing else is known about the client. It is a simple approach to save/allocate 4% of your income per year to retirement saving accounts. Inflation erodes your savings as the value of the dollar decreases so saving 4% with a 3% inflation does not yield an actual 4% saved since your dollar will be worth much less years late when you do retire.

Seth, who is about 55 years old, runs a local Po-Boy shop in New Orleans. He has several high school kids who work for him part time. Seth's mom, Robin, helps make the best meatballs in the universe and works there full time. Which of the following plans makes the most sense for Seth if he earns about $120,000 and does not want to spend too much on a retirement plan?

401(k) / profit sharing plan.

Generally, which of the following are contributory plans?

401(k) and thrift plans. Rationale Employers generally contribute to money purchase pension plans, ESOPs, and profit sharing plans. Employees contribute (thus contributory plans) to 401(k)s and thrift plans.

NOTCM partnership has 5 partners who have entered into a binding buy/sell agreement that requires any surviving partners to purchase the partnership interest of any partner to die. The partnership uses an entity approach to fund this arrangement. How many insurance policies are required to satisfy this arrangement?

5. Rationale The entity approach requires one insurance policy for each of the five partners

Jasmine is 53-years old and earns $115,000 a year. She saves 12% of her annual gross income for retirement. Jasmine will pay off her mortgage by the time she retires. Her monthly principal and interest payment is $1,950.21. Calculate Jasmine's wage replacement ratio using the top-down approach (round to the nearest %) and using pre-tax dollars. Assume that she wants to maintain her lifestyle.

68%

Ralph, a 40 year old nurse who earns $80,000 a year, saves 14% of his annual gross income. Assume that Ralph wants to maintain his exact pre-retirement lifestyle. Calculate Ralph's wage replacement ratio using the top-down approach and do not include the impact of income taxes. (NOTE: include savings rate as well as relevant payroll/FICA tax).

78%

Jenny, age 51, earns $300,000 annually from Infinity, which sponsors a 401(k) profit sharing plan, and matches all employee deferrals 100% up to a 3% contribution. What is the maximum total contribution to Jenny's account in 2018, including both employee and employer contributions?

9,000

Which of the following is true regarding QDROs?

A QDRO distribution is not considered a taxable distribution if the distribution is deposited into the recipient's qualified plan. Rationale The plan document, not the court, determines how the QDRO will be satisfied. No particular form is required for a QDRO, although some specific information is required. Form 2932-QDRO is not a real form. QDRO distributions may be subject to the 10% early withdrawal penalty if the distribution is not deposited into the recipient's qualified plan.

Westgate Inc., recently adopted a profit sharing plan. Westgate has two offices, the North Westgate office and the South Westgate office. There are 10 employees in the North Westgate office, 5 of which are eligible for the plan; and 15 employees in the South Westgate office that are all eligible for the plan. Which of the following statements is true?

A Summary Plan Description must be furnished to each participant within 120 days of plan establishment. Rationale Option a is incorrect because the employees must be notified by mail between 10 and 24 days before mailing the determination letter. Option c describes a master plan. Option d is incorrect because all employees must be notified if they work in an office where an eligible employee exists.

Brisco, now deceased, was married for 12 years. He had two dependent children, ages 10 and 12, who are cared for by their mother age 48. His mother, age 75, was his dependent and survived him. At the time of his death, he was currently but not fully insured under Social Security. His dependents are entitled to all of the following benefits except:

A parent's benefit. Rationale A lump-sum death benefit of $255 is payable to the surviving spouse or children of the deceased worker if he was fully or currently insured. The children's benefit is payable because Brisco was either currently or fully insured. It is 75% of his PIA. The children's mother would be entitled to a benefit for caring for the children under the age of 16. His dependent mother is only entitled to a benefit if he was fully insured, not currently insured.

Medicare Part A provides hospital coverage. Which of the following persons is not covered under Part A?

A person 62 or older and receiving railroad retirement. Rationale Medicare Part A requires a person to be age 65. People who are disabled or have permanent kidney failure are entitled to Medicare at any age.

All of the following statements concerning the Social Security system are correct except:

A special one-time payment of $1,000 may be made to a deceased worker's spouse or minor children upon death. Rationale A special one-time payment of $255 may be made to a deceased worker's spouse or minor children upon death.

Which of the following concerning the Social Security system is correct?

A worker who takes early retirement benefits will receive a reduced benefit because he will receive more monthly benefit payments as payments commence earlier than if the worker had waited and retired at full retirement age

Cheque Company has 100 eligible employees and sponsors a defined benefit pension plan. The company is unsure if they are meeting all of their testing requirements. How many employees (the minimum) must be covered by Cheque Company's defined benefit pension plan for the plan to conform with ERISA?

A) 40. Rationale The 50/40 rule requires that defined-benefit plans cover the lesser of 50 employees or 40% of all eligible employees. In this example, 40% of 100, or 40 employees, would be the lesser of these two amounts.)

All the following statements concerning prohibited transactions under ERISA are correct, EXCEPT:

Acquisition of employer securities by a pension plan is a prohibited transaction.

Kohler Company allows a 25% discount to all nonofficer employees. Officers are allowed a 30% discount on company products. Kohler's gross profit percent is 35%. Which of the following is true?

All discounts taken by officers (30%) are includible in their gross income because the plan is discriminatory. Rationale The plan is discriminatory to non-highly compensated employees; therefore, all discounts actually taken by officers are includible in the officers' income, not just the excess of what is available to the nonofficers. Any discount taken by a nonofficer would be excluded from the employee's gross income.

An employer may reduce the three percent matching contribution requirement for a calendar year in a SIMPLE, IRA, but only under which of the following circumstances? 1. The limit is reduced to no less than one percent. 2. The limit is not reduced for more than two years out of the five year period that ends with (and includes) the year for which the election is effective. 3. Employees are notified of the reduced limit within a reasonable period of time before the 60 day election period for a salary reduction agreement.

All of the above must be present. Rationale All of the circumstances listed must be present to permit the employer to reduce the three percent matching contribution requirement for a calendar year in a SIMPLE IRA according to IRC Section 408(p)(2)(C)(ii).

Which of the following are examples of fringe benefits that are not de minimus?

All of the above. Rationale All of the above are not de minimus

A worker's AIME:

All of the above. Rationale All of the above are correct. A)Must be determined by converting actual earnings into current dollars through an indexing factor. B) Is determined from wage information over prior years' work. C) Uses the highest 35 years of indexed earnings (for workers that worked at least that long). D) Yields an average amount of monthly earnings for all indexed years.

Distributions may be paid from a 403(b) account after:

All of the above. Rationale The other event not listed was, for salary reduction contributions, the employee endures a severe hardship according to IRC Section 403(b)(11). A) Employee death or disability. B) Employee turns age 59½. C) Employee is separated from service.

Phillip, who is currently age 52, made his only contribution to his Roth IRA in 2019 in the amount of $6,000. If he were to receive a total distribution of $11,000 from his Roth IRA in the year 2024 to purchase a new car, how would he be taxed?

Although Phillip waited five years, the distribution will not be classified as a "qualified distribution" and will therefore be taxable to the extent of earnings and will be subject to the 10% early distribution penalty on the amount that is taxable. Rationale A distribution from a Roth IRA is not includible in the owner's gross income if it is a qualified distribution or to the extent that it is a return of the owner's contributions to the Roth IRA. A qualified distribution is one that meets BOTH of the following tests: The distribution was made after a five-taxable-year period, and the distribution was made for one of the following reasons: • Owner has attained age 59½. • Distribution was made to a beneficiary or the estate of the owner on or after the date of the owner's death. • Distribution was attributable to the owner's disability. • Distribution was for a first-time home purchase. The 10 percent early withdrawal penalty under IRC § 72(t) applies to any distribution from a Roth IRA includible in gross income. The 10 percent early withdrawal penalty under IRC § 72(t) also applies to a nonqualified distribution, even if it is not then includible in gross income, to the extent it is allocable to a conversion contribution, and if the distribution is made within the five-taxable-year period beginning with the first day of the individual's taxable year in which the conversion contribution was made.

Which term or phrase best completes the following sentence: "Employee contributions in 403(b) plans are ______."

Always fully vested. Rationale Employee contributions in 403(b) plans are always fully vested, whereas employer contributions may be subject to a vesting schedule according to plan documents.

Individual accounts in 403(b) plans may not be in one of the following forms?

An account invested in individual stocks in companies in the S&P 500. Rationale Individual accounts in 403(b) plans must be options a, c, or d and thus, investing in individual stocks in S&P 500 companies is not allowed. Such stock may be owned through mutual funds only.

Split-dollar life insurance is:

An insurance arrangement in which the employer is the owner of the policy and is also the beneficiary to the extent of the premiums paid by the employer. Rationale The employer is usually the owner of a split-dollar policy and will be repaid the total of the premiums it has paid for the insurance.

Bobby Brown would not listen to his financial advisor and decided to rollover his qualified plan assets to a traditional IRA. Which of the following is correct?

Bobby has lost some of his creditor protection by moving the funds from a qualified plan to an IRA. Rationale Choice a is not correct because ten year forward averaging, pre-74 capital gain treatment and NUA treatment are available in a qualified plan, but not available in an IRA. Choice b is not correct because qualified plans can investment in life insurance and collectibles, which is not permitted in an IRA. Choice c is not correct, as he could have converted direct from a qualified plan to a Roth IRA. Choice d is correct as the assets are no longer protected under ERISA. The assets will be protected under bankruptcy law, but not ERISA.

Read the following statements: 1. Employee contributions to a qualified retirement plan are always 100 percent vested. 2. Employers are permitted to institute quicker vesting schedules than required by law. Which of these statements is/are true?

Both 1 and 2

Ricky receives stock options for 12,000 shares of XYZ Corporation with an exercise price of $10 when the stock is trading on the national exchange for $10 per share. The XYZ company plan is an Incentive Stock Option Plan. Which of the following statements are true regarding the options? 1. Ricky will be required to hold any ISOs for more than a year after exercise and more than two years from the grant date to have long-term capital gains. 2. 2,000 of the options are NQSOs.

Both 1 and 2. Rationale To the extent the fair market value of the stock for which the ISO is exerciseable for the first time during any calendar year exceeds $100,000, the excess is treated as a nonstatutory stock option. Therefore, 2,000 of the options are NQSOs.

Which statements are generally correct regarding penalties associated with IRA accounts? 1. Distributions made prior to 59½ are subject to the 10% premature distribution penalty. 2. There is a 50% excise tax on a required minimum distribution not made by April 1 of the year following the year in which age 70½ is attained.

Both 1 and 2. Rationale Statements 1 and 2 are both correct.

Decatur 401(k) Plan maintains a loan program for its participants. The plan has 50 participants, three of whom had participant loans. Decatur conducted a year-end review of its loan program and found the following: • Bob received a loan from the plan one year ago for $60,000 over a five-year term, amortized monthly using a reasonable interest rate. Bob timely made the required payments. Bob's vested account balance is $180,000. • Sandi received a loan of $10,000 to help her mother move to Florida this year, amortized over 72 months. Payments are timely and the interest rate is reasonable. Which of the following individuals have loans that do not comply with the IRC?

Both Bob and Sandi. Rationale Bob's loan exceeds the $50,000 limit and Sandi's exceeds the five-year rule.

Who can establish SIMPLE Plans

C Corp S Corp Limited Liability Companies Partnerships Proprietorships Gov. Entities (no annual filing costs) must earn at least $5,000 File 5035 SIMPLE form

Which of the following is an example of a qualified retirement plan? a. IRA b. 457 plan c. ESOP d. SEP IRA

C. ESOP

Which of the following statements is not correct regarding stock bonus plans? a. The required repurchase option for a stock bonus plan can create potential cash flow issues in the future. b. The eligibility for a stock bonus plan could be age 20 and six months of service. c. Stock bonus plans are generally as cost efficient to operate as profit-sharing plans. d. Stock bonus plans allow for the current deductibility of non-cash contributions by employers.

C. Stock bonus plans are generally as cost efficient to operate as profit-sharing plans.

In June 2019, Cody converts $100,000 in his 401(k) plan to a Roth account through an in-plan Roth rollover. The value of the assets in the Roth account drops by 40 percent due to a significant decline in the stock market that occurs in August 2019. The in-plan Roth rollover results in Cody incurring $100,000 of taxable income, when he could have waited and converted only $60,000 (after the 40 percent drop). Which of the following statements is correct?

Cody cannot recharacterize the conversion. Rationale In-plan Roth rollovers cannot be "undone" or recharacterized. The other choices are not correct.

Is collecting an employee census an important step in recommending a qualified plan for business? Explain in detail the rationale behind your answer.

Collecting employee census is important for gathering company information that will have an affect on the benefits specific qualified plans will give. Collecting the data on employees creates a better picture of who they would like to compensate the most to the least as well as assuring a high retention ratio if the benefits are correctly distributed according to data collected. Employee census's are vitally important to smaller business owners who would like to gain the most tax breaks, save money and provide sufficient benefits.

All the following statements concerning deductible IRA contributions are correct, EXCEPT:

Compensation includes alimony payments, separate maintenance payments, and investment income.

Which of the following statements concerning the Roth IRA is correct?

Contributions to a Roth IRA can be withdrawn at any time without incurring income tax.

The Good Company must permit which of the following employees to participate in their SEP?

Danielle who earned $650 of compensation for the year and has worked for Good Company for the last 10 years.

List 2 defined benefit plans and 2 defined contribution plans. Indicate with each plan whether it's a profit sharing plan or a pension plan.

Defined Benefit plans include: Defined benefit pension plan, Cash balance Pension Plan Defined Contribution Plans : Money Purchase plan and Target benefit plans (Both pension) Profit sharing DC: Stock Bonus, 401(k) Thrift Plan

What is the definition of defined benefit and defined contribution plans? List two examples of each.

Defined benefit plans are retirement investment vehicles that require employers to assume full risk and responsibility of contributions / benefits to be paid to employee. Examples include a cash benefit pension plan or an age base pension plan. Defined contribution plans can either be a pension plan or profit sharing plan which shift risk and responsibility of funds to the employee benefitting from the account. Examples include cash balance account or a 401(k) profit sharing plan where beneficiaries assume the risk of investing the funds.

Which of the following statements is true regarding C Corp Division?

Dividend distributions are taxed at the individual shareholder level is the true statement. As this is an income nature receipt of shareholder received from company and his income should be taxable in hand of taxpayer. Dividend is part of income which is distributed by company to his shareholders. So the dividend distribution is taxed at the individual shareholder level.

Which phrase best completes this sentence: "The exclusion for no-additional-cost services applies to any service provided by the employer to an employee that _____."

Does not cause the employer to incur any substantial additional costs or lose revenue. Rationale Option a is incorrect because the employer is losing revenue. Option b is incorrect because it is focused on the employer's cost, but it should weigh the employer's cost. Option d is also incorrect because no additional costs does not cause the "lose revenue" aspect of the test.

Dr. Wood has taught accounting at FSU (Florida Sate University) for the last 30 years and is expected to retire in a few years, at age 65. FSU sponsors a 403(b) plan and a 457(b) governmental plan. Both plans have Roth accounts available. She has been diligent and always contributed the maximum amounts to the traditional accounts in each of the plans. She is also a partner in an equipment business that has lost money the last several years and she has a net operating loss carry forward. Which of the following statements is true?

Dr. Wood is able to rollover amounts in the traditional deferral account for the 457 plan into the Roth account without terminating employment. Rationale Choice b is not correct as forfeitures cannot go into a 457 plan. Also, it is unlikely there are any forfeitures. Choice c is not correct as 403(b) plans cannot be invested in individual securities. Choice d is incorrect as the assets in a public 457 plan are protected by trust.

Which of the following is not an example of a qualified retirement plan?

ESPP

Marie, the sole shareholder in Marie's Pastries, is contemplating establishing a qualified plan. The corporation's employee census is as follows: Marie's Pastries Employee Census Employee Age Compensation Ownership Length of Service Marie 55 $200,000 100% 30 Years Cheryl 38 $45,000 0 20 years Jeff 42 $28,000 0 14 Years Ruby 34 $24,000 0 11 Years Total $297,000 100% The company experiences very low turnover. Marie, a long-time widow, has always treated the employees like her family and the company has experienced very low turnover. She would like to use the retirement plan to assist her in transferring ownership interest to the employees as she is ready to retire. She has a strong preference for avoiding and deferring taxes. She is opposed to mandatory funding and indifferent to integration. Which plan would be appropriate for Marie?

Employee stock ownership plan. Rationale An ESOP would be the most appropriate plan to meet Marie's objectives. The stock bonus plan would allow Marie to transfer stock, but would not assist her immediately in her retirement plans. The defined benefit and money purchase pension plan would require mandatory funding. The ESOP would provide her with tax benefits and a diversified portfolio because of her age.

Who generally makes elective deferrals to a 401(k) plan?

Employees only. Rationale Generally, 401(k) plans are funded from both employee elective deferrals and employer matching contributions and non-elective deferrals, but the elective deferrals come from the employee.

$10,409. Rationale $56,000 Schedule C net income - 3,956 (less 1/2 self-employment taxes at 15.3% x 0.9235) $52,044 Net self-employment income x 0.20 (0.25 ÷ 1.25) $10,409 Keogh profit sharing contribution amount

Employer only. Rationale Employees contribute to 401(k) plans and thrift plans. The government does not make contributions to plans, except when it is the employer.

Which of the following generally contribute to defined benefit plans, profit sharing plans, and money purchase pension plans?

Employer only. Rationale Employees contribute to 401(k) plans and thrift plans. The government does not make contributions to plans, except when it is the employer.

All of the following are advantages of the 401(k) plan except: Correct!

Employers can sponsor 401(k) safe harbor plans without committing to annual contributions and without creating a deferred liability.

Roger has a small convenience store with three employees. Each of the employees earns $10 per hour and they each work approximately 1,500 hours per year. His business is very successful. He recently considered adopting a defined benefit plan, but felt it was too expensive. He looked into a 401(k) plan, but thought it was too complicated. Finally, a few of his friends recommended a SIMPLE plan and told him about the basics of the plan. Which of the following statements from his friends is the most incorrect?

Employers that sponsor SIMPLEs must provide a matching contribution to employees who defer money into the SIMPLE. Rationale Choice a is true. The limit is 100, not 500, but the statement is correct. Choice b is true. SIMPLEs cannot be established if the company is maintaining another qualified plan or SEP. Choice c is true. While SIMPLEs do not have an age requirement, the statement is true. Choice d is false. Employers must offer a match or a non-elective contribution.

Which of the following statements regarding the tax treatment of an entity purchase buy-sell agreement is correct?

For an employer to receive tax-free death benefits, the Employee Notice and Consent requirements of Section 101(j) must be met. Rationale Section 101(j) of the tax code sets forth rules regarding employee notice and consent when a business entity wishes to purchase a life insurance policy in which the employee will be the insured and the business will be the owner and beneficiary.In order to retain the income tax-free nature of the death benefit, the business is required, before the contract is issued, to: Notify the employee in writing that the business (policyholder) intends to insure the employee's life, and include notice of the maximum face amount for which the employee could be insured at the time the contract is issued; and Obtain from the employee written consent to being insured under the contract and to continued coverage after the insured terminates employment; and Inform the employee, in writing, that the business (policyholder) will be a beneficiary of any proceeds payable upon the death of the employee. Answer choice B is incorrect because the premiums paid are not deductible. C is incorrect because a stock redemption agreement is an entity purchase agreement; therefore, the death benefit will be paid to the business entity. Answer choice D is incorrect because the surviving owners are not involved in an entity purchase agreement. Each owner is in contract only with the business entity, not with the other owners.

Steve is self-employed as a marketing consultant. He works primarily with start-up internet companies helping to develop corporate brand programs. Several years ago, he established a 401(k) profit-sharing plan and has accumulated $385,000 in the plan. Which of the following forms should he file to meet his compliance requirements?

Form 5500 EZ. Rationale He must file Form 5500 EZ since it is a one-participant plan and total assets exceed $250,000. If assets were below this threshold, he would not have to file the form.

A person receiving Social Security benefits under the age of 65 can receive earned income up to a maximum threshold without reducing Social Security benefits by the earnings test. Which of the following count against the earnings threshold?

Gambling winnings. Rationale Gambling winnings are not an exempt type of income. All the others are not earned income for Social Security purposes.

Answer these three questions below: a) When is the early withdrawal penalty applicable to traditional IRA? b) What is the penalty? c) List 2 exemptions to the penalty.

Generally, early withdrawal from an Individual Retirement Account (IRA) prior to age 59½ is subject to being included in gross income plus a 10 percent additional tax penalty. Med exceede 10% Educational Ex

Mary Jane received 1,000 NQSOs with an exercise price of $25 per share when the stock was $25 on the market. Two years from the date of grant Mary Jane exercises when the stock price is $102. At exercise, Mary Jane:

Has W-2 income of $77,000. Rationale Mary Jane will have W-2 income of the difference between the market price and the exercise price ($102 - $25 x $1,000 = $77,000). She will not have an AMT adjustment for the exercise of an NQSO.

Which of the following is/are allowable rollover(s)? I. Traditional IRA to SEP-IRA II. Public 457(b) to Traditional IRA III. Roth IRA to Traditional IRA IV. 403(b) to Traditional IRA

I, II, and IV

Which of the following advantages is (are) obtained from installation of a defined-benefit retirement plan? I. The plan will increase employees' take-home pay. II. The plan will help the employer to attract needed employees. III. The plan will minimize the employer's investment risk. IV. The plan will allow the employer to deduct contributions in the year made.

II and IV only

Mr. Reid was very active and loved to go skiing. Unfortunately, he was prone to skiing though the trees and did not believe in helmets. In March of 2019, he skied into a large pine tree that abruptly ended his life. At that time, his Roth IRA contained regular contributions of $10,000, first made in 2017, a conversion contribution of $40,000 that was made in 2016, and earnings of $10,000. He never made any distributions from his IRA. When he established this Roth IRA (his first) in 2016, he named each of his two children, Bill and Phil, as equal beneficiaries. Each child will receive one-half of each type of contribution and one-half of the earnings. Which of the following is true regarding a distribution after Mr. Reid dies?

If Bill immediately takes out all $30,000 from the Roth IRA, $5,000 of the distribution will be characterized as ordinary income, but he will not have to pay a penalty. Rationale The question first requires a determination of whether the distribution is a qualified distribution or not. While death is one of the two prongs, the five year rule as not been met. Thus, the distribution is not a qualified distribution. Each of the choices deals with a full distribution by the beneficiary, which makes the problem a bit easier. A full distribution will result in the earnings being taxable. The earnings represent 16.67% of the value of the account and thus, $5,000 will be taxable. No penalty is assessed after death of the owner.

Roger has a small convenience store with three employees. Each of the employees earns $10 per hour and they each work approximately 1,500 hours per year. His business is very successful. He recently considered adopting a defined benefit plan, but felt it was too expensive. He looked into a 401(k) plan, but thought it was too complicated. Finally, a few of his friends recommended a SIMPLE plan and told him about the basics of the plan. Which of the following statements from his friends is the most correct?

If Roger's salary is $500,000 and the SIMPLE used a match, then his match would generally equal $13,000. Rationale Choice a is correct - the compensation limit for qualified plans is not considered if the SIMPLE has a matching contribution. Thus, 3% times $500,000 equals $15,000, but the match is limited to the annual limit of $13,000 (2019). Choice b is false - SIMPLEs have 100% vesting. Choice c is false - a non-elective benefit equals 2%. Thus, the employees would receive $300 each ($15,000 x 2%). Choice d is false - He could establish a SIMPLE 401(k), but most often a SIMPLE IRA is established.

All of the following statements concerning Social Security benefits are correct except:

In order to obtain SSI benefits, an individual must be age 65 or older and must be disabled. Rationale In order to obtain SSI benefits, an individual must either be age 65 or older OR must be disabled.

Which of the following benefits provided by an employer to its employees is currently taxable to the employee?

Incidental personal use of a company car. Rationale Personal use of a company car is a taxable fringe benefit. All of the other employer fringe benefits listed may be excluded from the employee's gross income.

Identify at least four main assumptions necessary for capital needs/retirement funding analysis.

Inflation Rate Savings Rate Expected Portfolio Return Timing of Saving

Jennifer received 1,000 SARs at $22, the current trading price of Clippers, Inc., her employer. If Jennifer exercises the SARs three years after the grant and Clipper's stock is $34 per share, which of the following statements is true?

Jennifer will have W-2 income equal to $12,000.

Jennifer received 1,000 SARs at $22, the current trading price of Clippers, Inc., her employer. If Jennifer exercises the SARs three years after the grant and Clipper's stock is $34 per share, which of the following statements is true?

Jennifer will have W-2 income equal to $12,000. Rationale At the exercise of a SAR, the employee receives the difference between the fair market value and the exercise price as W-2 income. Thus, Jennifer has W-2 income equal to $12,000 [($34-$22)x1,000].

ProLife Inc. has a written adoption assistance program that pays adoption expenses including fees, attorney fees, and other normal expenses. ProLife paid adoption expenses for the following employees during 2019. Employee and Age Employee AGI Adoptee and Age Amount Paid Health of Adoptee Joe (34) $150,000 Cindylou (18) $14,080 Excellent James (34) $260,000 Randi (6) $14,080 Excellent Donna (34) $80,000 Brooke (3) $14,080 Excellent Connie (32) $100,000 Silky (5) $14,080 Excellent Which of the following have income inclusions resulting from the employer adoption plan?

Joe and James. Rationale Joe has adopted an 18-year-old whose health is excellent, so he must include the amount paid as income. An "eligible child" must be under the age of 18 or be physically or mentally incapable of caring for himself. James is over the phaseout ($251,160 for 2019) and must include the $14,080 in income.

Josh recently died at the age of 63, leaving a qualified plan account with a balance of $1,000,000. Josh was married to Kay, age 53, who is the designated beneficiary of the qualified plan. Which of the following is correct?

Kay can receive annual distributions over her remaining single-life expectancy, recalculated each year. Rationale Kay can receive distributions over her remaining single-life expectancy. A spouse beneficiary can recalculate life expectancy each year. Statement a is incorrect. She is not required to distribute the entire account within 5 years. Statement b is incorrect. Kay can wait until Josh would have been 70½ and begin taking distributions over her life expectancy. Statement c is incorrect. The distribution will not be subject to the early withdrawal penalty because the distributions were on account of death. Kay could also roll the account over to her own IRA and begin distributions when she attains age 70½.

For 2018, what is the maximum employee contribution for a 401(k) plan for those under 50? What is the catch-up for those 50 and over? What is the annual additions limit for those under 50?

Max 401(k) employee contribution under 50 : $18,500 Max Catch up over 50 : $55,000 Annual addition limit: $6,000

Mike was awarded 1,000 shares of restricted stock of B Corp at a time when the stock price was $14. Assume Mike properly makes an 83(b) election at the date of the award. The stock vests 2 years later at a price of $12 and Mike sells it then. What are Mike's tax consequences in the year of sale?

Mike has a long-term capital loss of $2,000. Rationale In the year of sale, Mike will have a long-term capital loss of $2,000 ($14,000 - $12,000) because his right to the stock vested. When 83(b) is elected, losses are permitted after the right to the stock has vested.

Describe how a Monte Carlo and sensitivity retirement analyses may be superior to the annuity method.

Monte Carlo and Sensitivity retirement analyses account for the effects various individual and economic variables could have on a client's retirement fund. Since these account for as many possible short falls in the planning process as possible they are superior to the annuity method which must rely on many important and risky estimations such as RLE. If the client lives past their predicted RLE, the annuity method does not provide any financial safety since the design is to have nothing after passing away.

Maria earns $120,000. She defers $10,000 into her 401(k) traditional account and $5,000 into her 401(k) Roth account. Which of the following statements is/are true? $120,000 of earnings is subject to FICA. Her W-2 wages subject to Federal income tax equals $110,000.

Neither 1 nor 2 option is correct

Professor Stabler has one child, Benson, who is 18 years old and a full-time student at Disc University, a private university where Professor Stabler is the chairman of the Finance Department and a full-time employee. The cost of undergraduate tuition at Disc University is $15,000 per semester, but the children of all full-time employees may attend Disc University for free. Last semester Benson took a Russian history class that was oversubscribed. Twenty-five students were on the waiting list, but Benson was number two. Three students got into the full class. Which of the following are correct? 1. Professor Stabler has to include the value of the tuition remission in his income for last semester. 2. Professor Stabler would not have to include the value of the tuition remission in his income if it was a graduate program that Benson was enrolled in.

Neither 1 nor 2. Rationale There are no requirements that there is space available for the tuition reduction exclusion. This exception applies only to education below the graduate level.

Mary, age 50, has an IRA with an account balance of $165,000. Mary has recently been diagnosed with an unusual disease that will require treatment costing $50,000, which she will have to pay personally. Mary's AGI will be $100,000 this year. Which of the following statements are true? 1. Mary can immediately borrow up to $50,000 from her IRA account and repay the loan within five years. 2. Mary can distribute $50,000 subject to income tax but not subject to the 10% penalty because it will be used to pay medical expenses.

Neither 1 nor 2. Rationale Statement 1 is incorrect because loans are not permitted from IRAs. Statement 2 is incorrect because only the portion of the medical expense that exceeds 10% of AGI (2017 TCJA: 7.5% for 2017 and 2018) is exempt from the 10% penalty ($50,000 - $10,000 = $40,000). However, if it were classified as a disability, then she could avoid the penalty on the entire distribution.

Which of the following statements are correct regarding assets reverting back to the sponsor or a qualified plan? 1. Under a merger, assets from a qualified plan can revert back to the plan sponsor without regard to the relationship between the value of the plan assets compared to the value of the obligations under the plan. 2. Any reversion of plan assets will always be subject to a 20% penalty.

Neither 1 nor 2. Rationale Statement 1 is incorrect because the assets must exceed the plan liabilities. Statement 2 is incorrect because the penalty may be 20 percent or 50 percent.

Brenda, age 53 and a recent widow, is deciding between taking a lump-sum distribution from her husband's pension plan of $263,500 now or selecting a life annuity starting when she is age 65 (life expectancy at 65 is 21 years) of $2,479 per month. Current 30-year Treasuries are yielding 6 percent annually. Which of the statements below are true? 1. If she takes the lump-sum distribution, she will receive $263,500 in cash now and be able to reinvest for 34 years, creating an annuity of $4,570 per month. 2. If she takes the lump-sum distribution she will be subject to the 10% early withdrawal penalty.

Neither 1 nor 2. Rationale Statement 1 is false. She will only receive $210,800 ($263,500 less 20% withholding). Statement 2 is also false. The distribution is on account of death, an exception to the 10% early withdrawal penalty rule.

Tom, age 39, is an employee of Star, Inc., which has a profit sharing plan with a CODA feature. His total account balance is $412,000, $82,000 of which represents employee elective deferrals and earnings on those deferrals. The balance is profit sharing contributions made by the employer and earnings on those contributions. Tom is 100 percent vested. Which of the following statements is/are correct? 1. Tom may take a loan from the plan, but the maximum loan is $41,000 and the normal repayment period will be 5 years. 2. If Tom takes a distribution (plan permitting) to pay health care premiums (no coverage by employer) he will be subject to income tax, but not the 10% penalty.

Neither 1 nor 2. Rationale Statement 1 is incorrect because he can take a loan equal to one-half of his total vested account balance up to $50,000. Statement 2 is incorrect because the exemption from the 10% penalty only applies to IRAs and only to the unemployed.

Mary, age 50, has an IRA with an account balance of $165,000. Mary has recently been diagnosed with an unusual disease that will require treatment costing $50,000, which she will have to pay personally. Mary's AGI will be $100,000 this year. Which of the following statements are true? 1. Mary can immediately borrow up to $50,000 from her IRA account and repay the loan within five years. 2. Mary can distribute $50,000 subject to income tax but not subject to the 10% penalty because it will be used to pay medical expenses.

Neither 1 nor 2. Rationale Statement 1 is incorrect because loans are not permitted from IRAs. Statement 2 is incorrect because only the portion of the medical expense that exceeds 10% of AGI (2017 TCJA: 7.5% for 2017 and 2018) is exempt from the 10% penalty ($50,000 - $10,000 = $40,000). However, if it were classified as a disability, then she could avoid the penalty on the entire distribution.

Ginger, who is 75 years old, requested from the IRS a waiver of the 60-day rollover requirement. She indicated that she provided written instructions to her financial advisor that she wanted to take a distribution from her IRA and roll it over into a new IRA. Her financial advisor inadvertently moved the funds into a taxable account. Ginger did not make the request of the IRS until five years after the mistake was made. Will the IRS permit the waiver?

No. Ginger waited an unreasonable amount of time before filing the request. Rationale The IRS generally grants such requests if timely made. However, Ginger should have realized this long before five years. She would have reported interest on her Form 1040 which would have caused her to realize the mistake. She certainly would have received account statements. Choice a is false. Choice b would be correct if Ginger had filed the request timely. Choice c is false and there is no such one-year period.

In 2019, Chip, an accomplished professional race car driver, is to receive a signing bonus for agreeing to drive for Hot-Lap International, a racing team. Hot-Lap agrees to establish a NQDC agreement with Chip to defer the bonus beyond Chip's peak income producing years. Hot-Lap transfers the bonuses to an escrow agent, subject to the risk of forfeiture to team creditors in bankruptcy, who invests the funds in securities acting as a hedge against inflation. The bonus is deferred until 2020 and is then paid to Chip in years 2020-2029. When is the income deductible by the employer and includible by Chip? Option Employer Deduction Employee Inclusion A 2019 2019 B 2020 2020 C 2020-2029 2020 D 2020-2029 2020-2029

Option D. Rationale The income is only deductible when includible by Chip in 2020-2029.

What is the PBGC and what plans are covered by the PBGC? What is the maximum guaranteed benefit for 2018 for retirees at age 65?

PBGC is the Pension Benefit Grantee Corporation. Is the governmental agency founded by ERISA and insures all qualified pension plans set up by private companies for their employee's retirement funds. They insure each employee up to $55,000 (max benefit) when they have reached the retirement age of 67.

Describe what the following Medicare parts cover: 1) Part A 2) Part B 3) Part C 4) Part D

Part A : Hospital Insurance Part B : Medical Insurance ( doctor visits out patient services) Part D : Prescriptions Part C is Medicare Advantage, the HMO plan for Medicare that replaces the other three.

HMO Inc. is paying the premium for long-term care policies for their 250 employees. How are these payments treated for federal income tax purposes?

Payments for group premiums for long-term care are tax deductible to the employer and not taxable income to the employee. Rationale Employer payments for the group premiums are tax deductible to the employer and not taxable income to the employee.

Andrea recently died at age 77, leaving behind a qualified plan worth $200,000. Andrea began taking minimum distributions from the account after attaining age 70½ and correctly reported the minimum distributions on her federal income tax returns. Before her death, Andrea named her granddaughter, Reese age 22, as the designated beneficiary of the account. Now that Andrea has died, Reese has come to you for advice with respect to the account. Which of the following is correct?

Reese can roll the account over to an IRA and name a new beneficiary. Rationale Statement a is incorrect because the five-year rule only applies if there is no designated beneficiary, or if a charity is the beneficiary. Since minimum distributions had already begun at the time of Andrea's death, Reese must take distributions over the greater of Reese's life expectancy or Andrea's life expectancy. Thus, statement c is incorrect because Andrea's life expectancy is definitely shorter than Reese's life expectancy. While statement b may appear to be correct, Reese would need to take distributions from the account based on her remaining life expectancy reduced by one year. She is not allowed to recalculate her life expectancy. Statement d is correct. However, the account must be in the name of the deceased (Andrea) for the benefit of Reese. In addition, the distributions can be taken over Reese's single life expectancy and not over a joint life expectancy.

Conversion to a Roth IRA from a Traditional IRA:

Requires a tax paid upon conversion.

Medical Trials Inc. has a cafeteria plan. Full-time employees are permitted to select any combination of the benefits listed below, but the total value received by each employee must be $6,500 a year or less. 1. Group medical and hospitalization insurance for employee only, $3,600 a year. 2. Group medical and hospitalization insurance for employee's spouse and dependents, $1,200 additional a year. 3. Child-care payments, actual cost not to exceed $5,000. 4. Cash required to bring the total of benefits and cash to $6,500. 5. Universal variable life insurance $1,000.

Robin chooses 1 and 2 and $1,700 cash. Robin must include $1,700 in taxable income. Rationale Option d is correct because cash must be included in income. Option a is incorrect because the entire cash distribution will be taxable. Option b is incorrect because the universal variable life insurance premiums of $1,000 cannot be excluded from Matt's gross income. Option c is incorrect because child care payments are excludable benefits.

Which retirement plan has the latest deadline for it to be set up for a plan year?

SEP-IRA

2018 Contribution Numbers

SIMPLE Elective C. - $12,500 Over 50 Catch up: $3,000 403(b) Annual Def: $18,500 Catchup over 50: $6,000 457 Plans Annual Def: $18,500 Catchup over 50: $6,000 Final 3 year Cath up: $18,500

Which of the following concerning the Social Security system is correct?

SSI benefits are funded by the Treasury, not Social Security taxes, as are the other benefits. Rationale The Social Security retirement benefit is payable at full retirement age with reduced benefits as early as age 62 to anyone who has obtained at least a minimum amount of Social Security coverage. The two Medicare trust funds are the federal Hospital Insurance Trust Fund for Part A and the Supplementary Medical Insurance Trust Fund for Part B of Medicare benefits. Social Security benefits can be paid to the dependent parents of a deceased insured worker at age 62 or over.

What are the provisions of a safe harbor 401(k) plan? Be specific about employer contributions with the two safe harbor options.

Safe Harbor 401k Plan options are either: 1. Contributory- Basic match - 100% match on the first 3% of deferred compensation plus a 50% match on deferrals between 3% and 5% (4% total). . 2. Non-Contributory- Enhanced match - Must be at least as much as the basic match at each tier of the match formula. A common formula is 100% match on the first 4% of deferred compensation. Safe harbor nonelective contribution - 3% (or more) of compensation, regardless of 401(k) deferrals. where the employee is responsible for their own contributions without employer assistance.

Social Security is funded through all of the following except:

Sales tax. Rationale Employee and employer payroll tax and self-employment tax are the sources of funding for Social Security. Sales tax does not fund Social Security.

In May 2019, Seth converts $100,000 in his traditional IRA to a Roth IRA. The value of the assets in the Roth IRA drops by 40% due to a significant decline in the stock market that occurs in October 2019. The Roth conversion results in Seth incurring $100,000 of taxable income, when he could have waited and converted only $60,000 (after the 40% drop). Which of the following statements is correct?

Seth cannot recharacterize the conversion. Rationale Prior to 2018, taxpayers had the ability to recharacterize a Roth conversion up to the due date of the income tax return, including extensions. As a result of The TCJA 2017, Roth conversions cannot be recharacterized after 2017.

Laura, age 43, has several retirement accounts and wants to know what accounts can be rolled over to other accounts. Which of the following statements regarding rollovers is not correct?

She could rollover her traditional IRA to her SIMPLE IRA. Rationale Choices a, b and c are all correct and permissible. Basically no other retirement assets can be rolled over to a SIMPLE IRA except assets currently in a SIMPLE IRA.

Cindy Sue has been with CS Designs, Inc. for five years. CS Designs has a deferred compensation plan to provide benefits to key executives only. CS Designs contributed $400,000 into a trust for Cindy Sue's benefit under the company's deferred compensation plan. The plan requires that executives must work for the company for 10 years before any benefits can be obtained from the plan. Cindy Sue has come to you to determine when she will be subject to income tax on the contribution by the employer. Which of the following is correct?

Since Cindy Sue cannot receive the benefits until she has been with the employer for 10 years, the substantial risk of forfeiture doctrine will not require inclusion in income for the current year contributions made by the employer. Rationale The economic benefit and constructive receipt doctrines will not cause inclusion because the assets are forfeitable if she does not stay the required length of service. Deferred compensation plans are by nature discriminatory. The contributions will be included if the employee has an economic benefit, no risk of forfeiture, or constructive receipt.

Steve, age 69, is an employee of X2, Inc. He plans to work until age 75. He currently contributes 6 percent of his pay to his 401(k) plan, and his employer matches with 3 percent. Which one of the following statements is true?

Steve is required to take minimum distributions from his 401(k) plan beginning April 1 of the year after he retires. Rationale Generally, an individual must receive his or her first minimum distribution by April 1 following the year the individual attains age 70½. However, if the individual remains employed beyond age 70½, he or she may defer minimum distributions until April 1 of the year following the year of retirement. This exception to the general rule only applies to the employer's qualified plan. Therefore, answers a and c are incorrect. Answer d is incorrect because Steve can continue to contribute to the 401(k) plan as long as he is still working for X2, Inc. and the plan permits.

Company A has been capitalized by MJBJ Vulture Capital, a venture capital company. Company A's cash flows are expected to fluctuate significantly from year to year, due to phenomenal growth. They expect to go public wthin three years. Which of the following would be the best qualified plan for them to consider adopting?

Stock bonus plan. Rationale A stock bonus plan will allow equity participation without the use of cash flows and the public offering will eventually provide liquidity.

Patrick works for the Atlanta Falcons in the marketing deparment. The Falcons have an athletic facility on the premises for employees other than football players. The value provided by the Falcons to Patrick of this fringe benefit is not included in Patrick's gross income if:

Substantially all of the use of the facility is to be employees of the Falcons, their spouses or their dependent. Rationale Option b is incorrect because the employer must operate the "on premises" athletic facility. Option c is incorrect because the facility must be "on the premises."

Which of the following are correct?

Surviving spouses are entitled to 100 percent of the worker's benefit amount after the worker dies. Rationale Option a is correct. Option b is wrong because spouses have to be married 10 years, not 8. Option c is wrong because the age is closer to 82, not 92.

Employer-sponsored life insurance is usually referred to as group life insurance. Which type of life insurance (offered as group life) is beneficial to both the employer and the employee from a tax standpoint?

Term life insurance. Rationale Group term life insurance premiums are deductible by the employer and excludable from income by the employee (up to $50,000 of death benefit). Group term provided by an employer in excess of $50,000 causes W-2 inclusion to the employee utilizing the Section 79 Schedule.

The beneficiary of key person life insurance is usually:

The company. Rationale The beneficiary is usually the company because the company has suffered the loss.

Roger converted all $100,000 in his traditional IRA to his Roth IRA on December 1, 2015 (his first Roth contribution or conversion). His Form 8606 from prior years shows that $20,000 of the amount converted is his basis. Roger included $80,000 ($100,000 - $20,000) in his gross income on his Form 1040 for the year. On April 5th, 2019, Roger made a regular contribution of $5,000 to a Roth IRA for the 2018 year. Roger took a $10,000 distribution from his Roth IRA on July 31st of 2019 to purchase a ticket for a trip on a cruise ship for his 61st birthday present to himself. How is the distribution taxed if the value of the account just before the distribution equals $120,000?

The distribution is tax free and penalty free. Rationale The question first requires a determination of whether the distribution is a qualified distribution or not. It is not a qualified distribution. Roger is over the age of 59½ but does not meet the five year rule. Therefore, the distribution first comes out of contributions, then conversions, then earnings ($5,000 from contribution and $5,000 from conversion). Since he is over the age of 59½, there is no penalty assessed. The entire distribution is tax free and not subject to a penalty.

Plans that require mandatory funding are generally funded by?

The employer. Rationale Plans that have mandatory funding features (defined benefit pension plans, cash balance pension plans, target benefit pension plans, money purchase pension plans) are generally funded by the employer.

Which statements are generally correct regarding penalties associated with IRA accounts? 1. Distributions made prior to 59½ are subject to the 10% premature distribution penalty. 2. There is a 50% excise tax on a required minimum distribution not made by April 1 of the year following the year in which age 70½ is attained.

The form tracks basis for contributions and conversions to Roth IRAs. Rationale Statement a is not correct because Form 8606 does not track basis for contributions to Roth IRAs. The taxpayer would have to determine the basis for Roth IRAs when completing Part IV of the form. The other statements are correct - it does track conversions to Roth IRAs, in-plan Roth rollovers, basis for nondeductible traditional IRAs and is used for distributions from Roth IRAs.

Which of the following statements is not correct about Form 8606?

The form tracks basis for contributions and conversions to Roth IRAs. Rationale Statement a is not correct because Form 8606 does not track basis for contributions to Roth IRAs. The taxpayer would have to determine the basis for Roth IRAs when completing Part IV of the form. The other statements are correct - it does track conversions to Roth IRAs, in-plan Roth rollovers, basis for nondeductible traditional IRAs and is used for distributions from Roth IRAs.

Pander's Box, a shop that specializes in custom trinket and storage boxes, has a 401(k) plan. The plan allows plan loans up to the legal limit allowed by law and they may be repaid under the most generous repayment schedule available by law. The plan has the following employee information: Employee 401(k) Balance Outstanding Loan Karen $400,000 $0 Teddy $250,000 $30,000 Josh $75,000 $0 Justin $15,000 $0 Which of the following statements is correct?

The maximum Justin can borrow from his account is $10,000. Rationale Justin can borrow one half of his vested account balance up to $50,000. Since the balance is below $20,000, he can borrow a full $10,000. State law does not require the repayment of the loans within a specified time; however, the plan can require that Teddy repay the loan immediately. Karen can only borrow one-half of her account balance up to $50,000, thus she can only borrow $50,000. Josh will not have to repay the loan in five years because the loan proceeds are being used for a home purchase and an extended period is available.

Reese has assets both in her Roth IRA and in her Roth account that is part of her employer's 403(b) plan. However, she is not sure about the differences between the two types of accounts. Which of the following statements would you tell her is correct?

The nature of the income received by beneficiaries in a qualified distribution is the same for distributions from both Roth IRAs and Roth accounts. Rationale Choice a is not correct because the five year holding period is separate for each type of account. Choice b is not correct because the Roth IRA has an additional distribution exception for first time home buying. Otherwise the rules are the same. Choice c is not correct because Roth IRAs do not have to comply with minimum distribution rules upon attainment of age 70½, while Roth accounts do have to comply.

Wallace and Associates is considering implementing a buy/sell agreement where each partner purchases a life insurance policy on each of the other partners. Which one of the following statements is correct given this information?

The partners are entering into a cross-purchase agreement. Rationale Statement a is incorrect because the entity approach occurs when the business entity purchases life insurance policies on each owner. Statement b is incorrect because the life insurance proceeds are tax-exempt. Statement c is incorrect because each policy will be for the deceased partner's share.

Which of the following is not true regarding profit sharing plans?

The plan is established and maintained by the individual employee.

Sew What, the best seamstress shop in town, sponsors a 401(k) plan. The plan provides a dollar-for-dollar match for employee contributions up to six percent and has immediate vesting for all contributions. For ADP purposes, the company has not made the top 20 percent election for the determination of who is highly compensated. The company has the following employee information:

The plan passes the ADP test if Joyce and Kali were not eligible.

Part B of Medicare is considered to be supplemental insurance and provides additional coverage to participants. Which of the following is true regarding Part B coverage?

The premiums for Part B are paid monthly through withholding from Social Security benefits. Rationale Only option b is correct. Option a is incorrect because participation can occur after the initial eligibility. Participation is not required to be maintained for life, and Part B does have deductibles and/or coinsurance.

Antoine immigrated from Italy last century, became a citizen and has worked the better part of his life in the United States, for which he is truly thankful. His full retirement age for Social Security benefits is age 66, but after a hard life working he wants to retire at age 63 and travel in America and back to his homeland. After contacting the Social Security administration, they informed him that his benefit at age 63 would be $1,200 per month. Just prior to retiring, he sold his business for $100,000. Which of the following statements is correct?

The sale of the business will not impact the amount of his retirement benefits from Social Security. Rationale Option b is false. The sale is not earned income. Therefore, it will not result in a reduction of benefits. Option c is false because it might not increase the portion of the benefit subject to taxation. 85 percent of his Social Security benefit may already be subject to taxation. Option d is false, as he would not receive Medicare until age 65.

Roger and Robin were happily married until Roger fell in love with Sam. As a result, Roger and Robin have agreed they need to get a divorce. As part of the process, the court has provided a domestic relations order that calls for Robin's profit-sharing plan to be divided into equal portions such that Roger will have his own account with half of the value of the retirement account. What type of approach has been taken?

The separate interest approach. Rationale The separate interest approach calls for splitting a retirement account into two separate accounts. Each party is free to act with regard to their separate account without the interference or consent of the other party. There is not such term as split payment approach or divided account approach.

One approach that is used in some domestic relations orders is to "split" the actual benefit payments made with respect to a participant under the plan to give the alternate payee part of each payment. Under this approach, the alternate payee will not receive any payments unless the participant receives a payment or is already in pay status. This approach is often used when a support order is being drafted after a participant has already begun to receive a stream of payments from the plan (such as a life annuity). This approach to dividing retirement benefits is often called what?

The shared payment approach. Rationale This is the definition of the shared payment approach. There is not such term as split payment approach or divided annuity approach.

Why are ISOs better than NQSOs for employees? (extra credit if you can provide two compelling answers).

There is no holding requirement in regards to employees receiving an ISO, is not tied to the performance of the company and not taxable if given the 83(b) election although employers cannot deduct these amounts given to employees. Whereas a NQSO's are tied with the performance to the company stock and gains on the stock will be taxed as ordinary capital gains tax. With ISOs you don't owe any ordinary income tax if it's a qualified sale.

Give two similarities and two differences between the traditional IRA and Roth IRA.

Traditional IRA contributions are tax-deductible on both state and federal tax returns for the year you make the contribution; withdrawals in retirement are taxed at ordinary income tax rates. Roth IRAs provide no tax break for contributions, but earnings and withdrawals are generally tax-free. Traditional IRAs require you to start taking required minimum distributions (RMDs)—mandatory, taxable withdrawals of a certain percentage of your funds—at age 70½, whether you need the money at that point or not. Roth IRAs, on the other hand, don't require any withdrawals during the owner's lifetime. SAME: If you are under 59½, you can withdraw up to $10,000 of Roth earnings penalty-free to pay for qualified first-time home-buyer expenses,

Joe Bob receives stock options (ISOs) with an exercise price of $18 when the stock is trading at $18. Joe Bob exercises these options two years after the date of the grant when the stock price is $39 per share. Which of the following statements is correct?

Upon exercise Joe Bob will have no regular income for tax purposes. Rationale Joe Bob does not have income at the date of exercise. Joe Bob's adjusted basis will be $18. The AMT income is equal to the difference between the fair market value and the exercise price ($39 - $18 = $21).

Which of the following cannot be held in an IRA account as an investment?

Variable life insurance. Rationale Life insurance is not permitted in IRA accounts. All of the other choices are permissible.

What is the definition of vesting? Why is it in the employer's interest to offer vested benefits?

Vesting is the security of funds/ benefits to be paid out to beneficiary at predetermined time. Vested benefits attracts employees to stay at the company for the security of steady & guarantied benefits if they remain at the company. Therefore employers can use vesting to decrease their employee turnover and reduce costs of searching for another.

What is the formula for the wage replacement ratio? Describe the difference between the top-down and bottom-up approach.

WRR= RWLE/RLE Top down approach uses a percentage of income and common sense to estimate the sum needed for retirement. It is a much faster estimation model but much less accurate than the Bottom up approach. Since the bottom up approach requires creating an estimated budget to create a detailed forecasting model to include a more individualized approach to estimating but also more time consuming. The Top down approach may be fine for younger individuals who have a long RWLE but should use the bottom up approach for those approaching retirement ages. Top down uses income less payroll taxes, income taxes, and savings to estimate WRR.

Life insurance used to fund a three person partnership buy/sell agreement using a cross purchase technique:

Will require six policies.

Tim, a participant in the Zappa retirement plan, has requested a second plan loan. Tim's vested account balance is $70,000. He borrowed $30,000 ten months ago and still owes $20,000 on that loan. Could he increase his maximum permissible loan if he repaid the outstanding loan before taking the new loan?

Yes. Paying off the loan will increase the loan available by $5,000. Rationale He can borrow the lesser of $50,000 or half of the vested account balance. The $50,000 must be reduced by the highest outstanding balance ($30,000) in the last twelve months, which equals $20,000. Half of the vested account balance ($70,000) less the outstanding loan of $20,000 equals $15,000. If the loan of $20,000 is repaid, which it could be, then the available loan would increase by $5,000 to $20,000.

MaryAnn, who is 75 years old, requested from the IRS a waiver of the 60-day rollover requirement. She indicated that she provided written instructions to her financial advisor that she wanted to take a distribution from her IRA and roll it over into a new IRA. Her financial advisor inadvertently moved the funds into a taxable account. MaryAnn did not make the request of the IRS until six months after the mistake was made. Will the IRS permit the waiver?

Yes. The mistake was the fault of the financial advisor and the IRS regularly grants waivers in these circumstances. Rationale The IRS generally grants such requests if timely made.

How does retiring early or retiring late in reference to their Normal Retirement Age affect the calculation of Social Security benefits? Be specific about the impact on benefits.

You get 8 percent for each year you delay beyond FRA. Retiring early can be done as early as age 62, but will reduce total benefits by either 1. 5/9 of 1% for the first early 36 months and 5/12 of 1% any other months earlier after the 36 month limit. or if you are born in 1950 or after there is a 30% reduction in SS benefits. Although it depends on each individual's RLE to decide if early withdrawal of benefits would be beneficial. Holding off Social Security retirement benefits will increase your benefit amounts per month by increasing your AIME for retirement.

What is a new comparability (new comp) plan? Is it a pension plan or a profit sharing plan? Is it a defined benefit or defined contribution plan? What sets it apart from other plans? Extra credit: Name at least one of the minimum allocation gateway tests for NHCEs for a new comparability plan to meet the nondiscrimination rules.

Your Answer: A new comparability plan is a p plan designed to allow the company to allocate contributions to employees based on classifications. It is a defined benefit plan that sets itself apart due to the allowance to skew contributions to higher compensated employees by using Social security benefits as a counted benefit in an employee's retirement fund thus allowing the company money to be allocated to others. The company is held to standards that require at least 5% contributions are consistently allocated to non-highly compensated employees ignorer to meet non discretionary rules.

Baily owns and operates Ben's Red Truck Shop (BRT), which is a sole proprietorship. She has self-employment income of $150,000. How much self-employment tax does she owe for 2018? a. $19,939. b. $20,271. c. $22,194. d. $22,950.

a. $19,939. Self-employment income $150,000 Times 92.35% A $138,525 Max Social Security Wage Base $128,400 Net $128,400 B Smaller of A or B $128,400 OASDI (amount above times 12.4%) $15,921.60 A times 2.9% $4,017.23 Total $19,938.83

Acme Inc. has 200 total employees, 150 of which are nonexcludable employees. Ten employees are highly compensated. Seven of the 10 highly compensated and 100 of the 140 nonhighly compensated employees are covered under Acme's qualified plan. The average accrued benefits for the highly compensated is 3% and the average accrued benefit for the nonhighly compensated is 1.5%. Which of the following statements is true regarding coverage? 1. The plan passes the ratio percentage test. 2. The plan passes the average benefits test. a. 1 only. b. 2 only. c. Both 1 and 2. d. Neither 1 nor 2.

a. 1 and 2 The ratio percentage test compares the % of nonhighly compensated to the % of highly compensated covered. The ratio must be greater than or equal to 70% for the plan to pass the ratio percentage test. The calculation for Wanka's qualified plan is as follows: NHC = 100 / 140 = 71.43% Pass HC 7 / 10 = 70% 71.43% / 70.00% = 102% Pass Acme's plan passes the ratio percentage test requirement of 70%. The average benefits test requires the average benefit of the nonhighly compensated employees to be at least 70% of the average benefit of the highly compensated. Acme's plan does not satisfy the average benefits test because the average benefit of the nonhighly compensated compared to the average benefit of the highly compensated is less than 70% (1.5%/3% = 50%) (fail).

Colin is 40 years old and wants to retire in 27 years. His family has a history of living well into their 90s. Therefore, he estimates that he will live to age 95. He currently has a salary of $150,000 and expects that he will need about 75% of that amount annually if he were retired. He can earn 8 percent from his portfolio and expects inflation to continue at 3 percent. Some years ago, he worked for the government and expects to receive an annuity that will pay him $20,000 in today's dollars per year beginning at age 67. The annuity includes a cost of living adjustment, which is equal to inflation. Colin currently has $200,000 invested for his retirement. His Social Security benefit in today's dollars is $30,000 per year at normal age retirement of age 67. How much does he need to accumulate at age 67 exclusive of his pension and Social Security benefits? a. $2.1 million. b. $2.2 million. c. $2.8 million. d. $2.9 million.

b. $2.2 million. The pension can be treated in the same manner as Social Security because it occurs at the same time and includes a COLA. Choice a is calculated as an ordinary annuity. Choice C is calculated without the pension and as an ordinary annuity. Choice d is calculated without the pension.

Which of the following vesting schedules may a top-heavy qualified cash balance plan use? a. 2 to 6 year graduated. b. 3-year cliff. c. 3 to 7 year graduated. d. 5 year cliff.

b. 3-year cliff. As a result of the PPA 2006, cash balance plans must vest at least as fast as a three year cliff vesting schedule. They cannot use the 2 to 6 year graduated schedule.

Tidewater Company has 1,000 eligible employees and sponsors a defined benefit pension plan. The company is unsure if they are meeting all of their testing requirements. How many employees (the minimum) must be covered by Tidewater's defined benefit pension plan for the plan to conform with ERISA? a. 40. b. 50. c. 400. d. 500.

b. 50. The 50/40 rule requires that defined-benefit plans cover the lesser of 50 employees or 40% of all eligible employees. In this example, 50 employees is less than 40% of 1000, or 400 employees.

Which of the following plans needs an actuary on an ongoing basis? a. Tandem plan. b. Cash balance plan. c. 412(e) plan. d. Target benefit plan.

b. Cash balance plan. The tandem plan consists of a money purchase pension plan and a profit sharing plan. The cash balance plan needs an actuary. The 412(e) plan is fully funded with insurance and does not need an actuary. The target benefit plan does not need an actuary after it is set up.

Larry is 58 and wants to retire by age 65. He expects that he will live to age 95. He currently has a salary of $140,000 and expects that he will need about 72% of that amount annually if he were retired. He can earn 9 percent in his portfolio while he is working. However, he expects that he will only earn 7 percent in his portfolio during retirement because he will adjust his asset allocation so that his portfolio is more conservative. He expects inflation to continue at 3 percent. Larry currently has $800,000 invested for his retirement. His Social Security benefit in today's dollars is $24,000 per year, assuming a retirement of age 65. He just calculated what he needs to save each year and it is more than he can afford. Which of the following is his best alternative to achieve his retirement goals? a. Modify his portfolio during retirement to achieve an 8% rate of return instead of a 7% rate of return. b. Delay his retirement by 2 years. c. Reduce his expected needs by $10,000 in today's dollars. d. None of the above will help him achieve his goals.

b. Delay his retirement by 2 years. Choice a reduces his annual required savings to $9,310 per year. Choice b reduces his annual required savings to $1,583 without considering the increase in his Social Security benefit from waiting to collect until age 67. Choice c reduces his annual required savings to $3,748 per year.

Qualified plan documents can be written to accommodate plan forfeitures in different ways. Which of the following statements is correct? a. Stock bonus plans always allocate plan forfeitures to participants' accounts since it is a stock plan. b. Safe harbor 401(k) plans never allocate plan forfeitures of matching contributions to participants' accounts. c. Defined benefit plans (but not cash balance plans) have two options to deal with forfeitures. d. Employers can use the forfeited funds to pay other expenses of the business outside the qualified plan.

b. Safe harbor 401(k) plans never allocate plan forfeitures of matching contributions to participants' accounts. Choice a is incorrect as these plans can use either method to deal with forfeitures. Choice c is correct as safe harbor 401(k) plans will not have forfeitures since they are 100% vested. Choice c is incorrect because DB plans must reduce plan costs since there are no accounts. Choice d is incorrect as employers cannot use funds in a qualified plan for other purposes (except under very unusual situations and subject to a sizable penalty).

Sean will be retiring soon. All of the following expenditures could be eliminated in his retirement needs calculation except: a. The $2,200 per year he spends on his work suits and dress clothes. b. The $18,000 annual mortgage payment he makes that is scheduled to end seven years into retirement. c. The FICA taxes he pays each year. d. The $22,000 per year he contributes to his 401(k) plan.

b. The $18,000 annual mortgage payment he makes that is scheduled to end seven years into retirement. Sean cannot eliminate his mortgage since it will not be paid off at retirement. He can eliminate the other expenses.

Qualified plans must satisfy many tests to maintain qualified status. Which of the following is correct regarding coverage tests? a. Defined benefit plans must pass any two of the four coverage tests. b. Coverage testing includes employees covered under a collective bargaining agreement as part of the calculation. c. Stock bonus plans must satisfy one of three coverage tests only. d. Employees who do not meet the eligibility requirements are still included in the determination of at least one of the coverage tests.

c. Stock bonus plans must satisfy one of three coverage tests only.

Parker is a highly skilled sales person at Byberry, which is a 30-year-old company that has grown significantly in terms of revenue and product offerings. It sponsors a pension plan that provides a benefit of 2% times the years of participation times the average of the final three years of salary. Parker has worked for Byberry for the last 30 years and earned $200,000 two years ago, $150,000 last year, and $250,000 this year. If he is retiring this year, how much should he expect to receive as a pension benefit? a. $200,000. b. $185,000. c. $160,000. d. $120,000.

d. $120,000. The average salary over the last three years is $200,000. The benefit equals 60 percent of $200,000 or $120,000.

Drake has worked for GT for the last 20 years and been a participant in its defined benefit plan. In the last ten years, his salary has increased significantly. Over the last ten years, his compensation was $300,000, $145,000, $200,000, $400,000, $225,000, $240,000, $233,000, $210,000, $150,000, and $290,000, respectively. In 2018, what is the most that he could receive as a pension payment? a. $330,000. b. $288,333. c. $275,000. d. $220,000.

d. $220,000. The maximum distribution from a defined benefit plan in 2018 is $220,000.

Parker is 50 and wants to retire in 15 years. His family has a history of living well into their 90s. Therefore, he estimates that he will live to age 95. He currently has a salary of $120,000 and expects that he will need about 65% of that amount annually if he were retired. He can earn 9 percent in his portfolio while he is working. However, he expects that he will only earn 7 percent in his portfolio during retirement. He expects inflation to continue at 3 percent. Parker currently has $350,000 invested for his retirement. His Social Security benefit in today's dollars is $30,000 per year at normal age retirement of age 67. His Social Security benefit will be reduced by 6 2/3 percent for each year he begins collecting before full age retirement. How much does he need to save each year to meet his retirement goals? a. $2,465. b. $2,987. c. $4,975. d. $6,855.

d. $6,855. Choice a is calculated using 9% as the earnings rate during retirement. However, the number is actually negative and not positive as indicated in the answer choice. Choice b is calculated without the adjustment for Social Security at age 65. Choice c is calculated as an ordinary annuity.

Jacob is a participant in JJ's defined benefit plan. Jacob is 37 years old and earns $160,000. He has 4 years of service for purposes of the plan and has worked at the firm for 5 years. The plan provides a benefit of 1.5% for each year of participation. The plan has the least generous vesting schedule possible. Almost 70 percent of the accrued benefits are attributable to the fifteen equal owners, who have all been working at the company for decades. If Jacob were to leave today, what percent of his salary (as defined by the plan) could he expect to receive at normal retirement? a. 3.6%. b. 4.8%. c. 6.0%. d. 6.4%.

d. 6.4%. This plan is top heavy based on the 70% of benefits for the equal owners - they each own more than 5% and are key employees. Therefore, vesting will shift and there is a minimum benefit that must be provided to non-key employees. His benefit will be 2% times YOS (8%) up to 20%. However, the benefit must be reduced based on his vesting. Five years of service represents 80% vesting. The benefit equals 6.4% (8% x 80%).

Defined benefits plans and cash balance plans are both pension plans. However, they are significantly different plans. Which of the following statements is true? a. A cash balance pension plan benefits younger and older employees equally. b. Both plans have hypothetical accounts for each plan participant. c. Both plan use the same vesting schedules whether the plan is top heavy or not. d. Both plans can provide for lump-sum benefits upon termination and/or full retirement age.

d. Both plans can provide for lump-sum benefits upon termination and/or full retirement age. Statement d is true. Statement a is false because cash balance pension plans usually benefit younger participants because of the guaranteed contribution rate and the guaranteed earnings rate. Statement b is false because a defined benefit plan promises a defined benefit at the participant's retirement. It does not have a hypothetical account like a cash balance plan. Statement c is false because cash balance plans must use the 3-year cliff vesting.

Marleen is a 52-year-old participant in the XYZ cash balance plan. She has been a participant in the plan for the last twenty years. XYZ, which has over 10,000 employees, is having financial difficulty and Marleen is concerned about the security of her pension. Which of the following is correct? a. The cash balance plan assets may include up to 25% of XYZ stock. b. The cash balance plan formula cannot be lowered in the future for current participants, but could be changed for new participants. c. The cash balance plan and its benefits are fully protected by the PBGC. d. Termination of the plan may affect vesting for some employees, but Marleen's vesting will not be impacted.

d. Termination of the plan may affect vesting for some employees, but Marleen's vesting will not be impacted. Choice a is incorrect because up to 10 percent of plan assets can be invested in the plan sponsor's stock. Choice b is incorrect as future benefits could be modified by the plan sponsor. Choice c is incorrect because the PBGC does not fully protect all benefits - only up to an annual limit. Choice c is correct. Choice d is incorrect as he has twenty years of service and benefits fully vest upon termination. Choice d is correct. Termination accelerates vesting, but based on Marleen's tenure, she is fully vested already.

Alfred has worked for CJD, a large manufacturer, for the last 20 years and is a participant in CJD's defined benefit plan. Alfred is concerned over the company's financial difficulties and is worried that management at CJD might modify his future benefits and cause his expected benefits at retirement to be reduced. Which of the following laws is designed to prevent that from occurring? a. Coverage b. Non-discrimination c. Anti-cutback d. There are no laws that prevent an employer from modifying future benefits, even in a defined benefit plan.

d. There are no laws that prevent an employer from modifying future benefits, even in a defined benefit plan. • Future benefits can be modified by employers. • Coverage is designed to ensure that a minimum number of non-highly compensated employees are benefiting under the plan. • Non-discrimination rules do not prevent loss of benefits. • Anti-cutback is a rule prohibiting a plan sponsor from amending a plan such that the accrued benefit of an employee is decreased or reduced by such amendment or change.

Which of the following statements are true? Defined benefit plans may use forfeitures to reduce plan costs or reallocate them to plan participants. If a prohibited transaction occurs in a qualified plan, a 100 percent penalty may be assessed if not timely corrected.

only 2


संबंधित स्टडी सेट्स

Mean, Median, Mode, and Confidence Intervals Sensitivity, specificity, and Predictive Value Standard Curves

View Set

Adult Development and Aging Exam 1

View Set

TXT Ch. 8 - Deformation and Strengthening Mechanisms

View Set

Skin Integrity and Wound Care Prep U Ex. 1

View Set

Tools Used By the Fed to Implement Monetary Policy

View Set

Chapter 33: Disorders of Renal Function - ML3

View Set